Sei sulla pagina 1di 125

Downloaded From : www.EasyEngineering.

net

ww
w.E
a syE
ngi
nee
rin
g.n
et

**Note: Other Websites/Blogs Owners Please do not Copy (or) Republish


this Materials, Students & Graduates if You Find the Same Materials with
EasyEngineering.net Watermarks or Logo, Kindly report us to
easyengineeringnet@gmail.com

Downloaded From : www.EasyEngineering.net


Downloaded From : www.EasyEngineering.net

S.NO CONTENTS Page.NO


UNIT I
RANDOM VARIABLES
1. Introduction 1
2. Discrete Random Variables 1
3 Continuous Random Variables 5
4 Moments 14
5 Moment generating functions 14
6 Binomial distribution 18
7 Poisson distribution 21
8 Geometric distribution 25
9
ww
10
11
Uniform distribution
Exponential distribution
Gamma distribution
27
29
31

w.E UNIT II
TWO –DIMENSIONAL RANDOM VARIABLES

asy
11 Introduction 37
12 Joint distribution 37
13 Marginal and Conditional Distribution 38
14
15
Covariance
En
Correlation Coefficient
43
44
16
17
Problems
Linear Regression gin 41
45
18
19 Problems ee
Transformation of random variables

UNIT III
rin
46
47

g.n
RANDOM PROCESSES
20 Introduction 49
21 Classification 50
22
23
24
stationary processes
Markov processes
Poisson processes
et 51
55
56
25 Random Telegraph processes 57

UNIT IV
CORRELATION AND SPECTRAL DENSITIES
26 Introduction 60
27 Auto Correlation functions 60
28 Properties 61
29 Cross Correlation functions 63
30 Properties 64
31 Power spectral density 65

Downloaded From : www.EasyEngineering.net


Downloaded From : www.EasyEngineering.net

32 66
Properties
33 Cross spectral density 66
34 Properties 67
UNIT V
LINER SYSTEMS WITH RANDOM INPUTS
35 Introduction 71
36 Linear time invariant systems 72
37 Problems 72
38 Linear systems with random inputs 73
39 Auto Correlation and Cross Correlation functions of inputs and 74
outputs
40 System transfer function 75

ww
41 Problems 76

w.E
asy
En
gin
ee rin
g.n
et

Downloaded From : www.EasyEngineering.net


Downloaded From : www.EasyEngineering.net

MA6451 PROBABILITY AND RANDOM PROCESSES LTPC3104

OBJECTIVES: To provide necessary basic concepts in probability and random processes for
applications such as random signals, linear systems etc in communication engineering.

UNIT I RANDOM VARIABLES 9+3 Discrete and continuous random variables – Moments –
Moment generating functions – Binomial, Poisson, Geometric, Uniform, Exponential, Gamma
and Normal distributions.

UNIT II TWO - DIMENSIONAL RANDOM VARIABLES 9+3 Joint distributions – Marginal and
conditional distributions – Covariance – Correlation and Linear regression – Transformation of
random variables.
UNIT III RANDOM PROCESSES 9+3 Classification – Stationary process – Markov process -

ww
Poisson process – Random telegraph process.
UNIT IV CORRELATION AND SPECTRAL DENSITIES 9+3 Auto correlation functions – Cross
correlation functions – Properties – Power spectral density – Cross spectral density –

w.E
Properties.
UNIT V LINEAR SYSTEMS WITH RANDOM INPUTS 9+3
Linear time invariant system – System transfer function – Linear systems with random inputs –

asy
Auto correlation and Cross correlation functions of input and output.
TOTAL (L:45+T:15): 60 PERIODS
OUTCOMES:

En
 The students will have an exposure of various distribution functions and help in acquiring
skills in handling situations involving more than one variable. Able to analyze the response of

gin
random inputs to linear time invariant systems.

TEXT BOOKS:

Reprint, 2007. ee
1. Ibe.O.C., “Fundamentals of Applied Probability and Random Processes", Elsevier, 1st Indian

rin
2. Peebles. P.Z., "Probability, Random Variables and Random Signal Principles", Tata McGraw
Hill, 4th Edition, New Delhi, 2002.

REFERENCES: g.n
India Pvt. Ltd., Bangalore, 2012.
et
1. Yates. R.D. and Goodman.D.J., "Probability and Stochastic Processes", 2nd Edition, Wiley

2. Stark. H., and Woods. J.W., "Probability and Random Processes with Applications to Signal
Processing", 3rd Edition,Pearson Education, Asia, 2002.
3. Miller. S.L. and Childers.D.G., "Probability and Random Processes with Applications to Signal
Processing and Communications", Academic Press, 2004.
4. Hwei Hsu, "Schaum‟s Outline of Theory and Problems of Probability, Random Variables and
Random Processes", Tata McGraw Hill Edition, New Delhi, 2004.
5. Cooper. G.R., McGillem. C.D., "Probabilistic Methods of Signal and System Analysis", 3rd
Indian Edition, Oxford University Press, New Delhi, 2012.

Downloaded From : www.EasyEngineering.net


Downloaded From : www.EasyEngineering.net

MA6451 PROBABILITY AND RANDOM PROCESSES

UNIT - I
RANDOM VARIABLES

Introduction

Consider an experiment of throwing a coin twice. The outcomes


{HH, HT, TH, TT} consider the sample space. Each of these outcome can be associated
with a number by specifying a rule of association with a number by specifying a rule of
association (eg. The number of heads). Such a rule of association is called a random
variable. We denote a random variable by the capital letter (X, Y, etc) and any particular
value of the random variable by x and y.

ww Thus a random variable X can be considered as a function that maps all elements in
the sample space S into points on the real line. The notation X(S)=x means that x is the
value associated with the outcomes S by the Random variable X.

w.E
1.1 SAMPLE SPACE
Consider an experiment of throwing a coin twice. The outcomes

asy
S = {HH, HT, TH, TT} constitute the sample space.

1.2 RANDOM VARIABLE


En
In this sample space each of these outcomes can be associated with a number by

gin
specifying a rule of association. Such a rule of association is called a random variables.
Eg : Number of heads

S = {HH, HT, TH, TT}


X(S) = {2, 1, 1, 0}
ee
We denote random variable by the letter (X, Y, etc) and any particular value of the
random variable by x or y.

rin
g.n
Thus a random X can be the considered as a fun. That maps all elements in the sample space S
into points on the real line. The notation X(S) = x means that x is the value associated with
outcome s by the R.V.X.
Example
In the experiment of throwing a coin twice the sample space S is et
S = {HH,HT,TH,TT}. Let X be a random variable chosen such that X(S) = x (the number of
heads).
Note
Any random variable whose only possible values are 0 and 1 is called a Bernoulli random
variable.

1.2.1 DISCRETE RANDOM VARIABLE


Definition : A discrete random variable is a R.V.X whose possible values consitute finite set of
values or countably infinite set of values.
Examples

Downloaded From : www.EasyEngineering.net


Downloaded From : www.EasyEngineering.net

MA6451 PROBABILITY AND RANDOM PROCESSES

All the R.V.’s from Example : 1 are discrete R.V’s


Remark
The meaning of P(X ≤a).
P(X ≤a) is simply the probability of the set of outcomes ‘S’ in the sample space for which
X(s) ≤ a.
Or
P(X≤a) = P{S : X(S) ≤ a}
In the above example : 1 we should write
3
P(X ≤ 1) = P(HH, HT, TH) =
4
3
Here P(X≤1) = means the probability of the R.V.X (the number of heads) is less than or equal
4

ww 3
to 1 is .
4
Distribution function of the random variable X or cumulative distribution of the random

w.E
variable X
Def :
The distribution function of a random variable X defined in (-∞, ∞) is given by

Note asy
F(x) = P(X ≤ x) = P{s : X(s) ≤ x}

En
Let the random variable X takes values x1 , x2 , ….., x n with probabilities P 1 , P 2 , ….., P n
and let x1 < x 2 < ….. <x n
Then we have
gin
F(x) = P(X < x1 ) = 0, -∞ < x < x,
F(x) = P(X < x1 ) = 0, P(X < x1 ) + P(X = x1 ) = 0 + p1 = p1

ee
F(x) = P(X < x2 ) = 0, P(X < x1 ) + P(X = x1 ) + P(X = x2 ) = p 1 + p 2
F(x) = P(X < x n) = P(X < x1 ) + P(X = x1 ) + ….. + P(X = x n )
= p 1 + p 2 + ………. + p n =1
rin
1.2.2 PROPERTIES OF DISTRIBUTION FUNCTIONS
Property : 1 P(a < X ≤ b) = F(b) – F(a), where F(x) = P(X ≤ x)
P(a ≤ X ≤ b) = P(X = a) + F(b) – F(a)
g.n
Property : 2
Property : 3
= F(b) – F(a) – P(X = b)
P(a < X < b) = P(a < X ≤ b) - P(X = b)
by prob (1) et
1.2.3 PROBABILITY MASS FUNCTION (OR) PROBABILITY FUNCTION
Let X be a one dimenstional discrete R.V. which takes the values
x1 , x2 , …… To each possible outcome ‘x i ’ we can associate a number p i .
i.e., P(X = xi ) = P(x i ) = p i called the probability of x i . The number
p i = P(x i ) satisfies the following conditions.

(i) p ( x i ) ≥ 0, ∀i (ii) ∑ p(x ) = 1
i =1
i

Downloaded From : www.EasyEngineering.net


Downloaded From : www.EasyEngineering.net

MA6451 PROBABILITY AND RANDOM PROCESSES

The function p(x) satisfying the above two conditions is called the probability mass
function (or) probability distribution of the R.V.X. The probability distribution {x i , p i } can be
displayed in the form of table as shown below.
X = xi x1 x2 ……. xi
P(X = xi ) = p i p1 p2 ……. pi

Notation
Let ‘S’ be a sample space. The set of all outcomes ‘S’ in S such that
X(S) = x is denoted by writing X = x.
P(X = x) = P{S : X(s) = x}
|||ly P(x ≤ a) = P{S : X() ∈ (-∞, a)}
and P(a < x ≤ b) = P{s : X(s) ∈ (a, b)}

ww
P(X = a or X = b) = P{(X = a) ∪ (X = b)}
P(X = a and X = b) = P{(X = a) ∩ (X = b)} and so on.

w.E
Theorem :1 If X1 and X2 are random variable and K is a constant then KX 1 , X 1 + X 2 , X 1 X 2 ,
K 1 X 1 + K2 X 2 , X 1 -X 2 are also random variables.

Theorem :2 asy
En
If ‘X’ is a random variable and f(•) is a continuous function, then f(X) is a random
variable.

Note
gin
If F(x) is the distribution function of one dimensional random variable then
I. 0 ≤ F(x) ≤ 1
II. If x < y, then F(x) ≤ F(y)
III. F(-∞) = lim F(x) = 0
x →−∞
ee rin
IV. F(∞) = lim F(x) = 1
x →∞
g.n
V.If ‘X’ is a discrete R.V. taking values x1 , x 2 , x3
Where x1 < x2 < x i-1 x i …….. then
P(X = xi ) = F(x i ) – F(xi-1 )
Example:1.2.1
et
A random variable X has the following probability function
Values of X 0 1 2 3 4 5 6 7 8
Probability P(X) a 3a 5a 7a 9a 11a 13a 15a 17a

(i) Determine the value of ‘a’


(ii) Find P(X<3), P(X≥3), P(0<X<5)
(iii) Find the distribution function of X.
Solution

Downloaded From : www.EasyEngineering.net


Downloaded From : www.EasyEngineering.net

MA6451 PROBABILITY AND RANDOM PROCESSES

Table 1
Values of X 0 1 2 3 4 5 6 7 8
p(x) a 3a 5a 7a 9a 11a 13a 15a 17a

(i) We know that if p(x) is the probability of mass function then


8

∑ p(x ) = 1
i =0
i

p(0) + p(1) + p(2) + p(3) + p(4) + p(5) + p(6) + p(7) + p(8) = 1


a + 3a + 5a + 7a + 9a + 11a + 13a + 15a + 17a = 1
81 a = 1
a = 1/81

ww
put a = 1/81 in table 1, e get table 2
Table 2

w.E X=x 0
P(x)
1 2 3 4 5
1/81 3/81 5/81 7/81 9/81 11/81 13/81 15/81 17/81
6 7 8

(ii) P(X < 3)


asy
= p(0) + p(1) + p(2)
= 1/81+ 3/81 + 5/81 = 9/81
(ii) P(X ≥ 3)
En
= 1 - p(X < 3)
= 1 - 9/81 = 72/81
(iii) P(0 < x < 5)
gin
= p(1) + p(2) + p(3) + p(4) here 0 & 5 are not include
= 3/81 + 5/81 + 7/81 + 9/81
3+5+7+8+9

81
ee
= ––––––––––––––– =
24
–––––

(iv) To find the distribution function of X using table 2, we get


81
rin
X = x F(X) = P(x ≤ x)
g.n
0

1
F(0)
F(1)
= p(0) = 1/81
= P(X ≤ 1) = p(0) + p(1)
= 1/81 + 3/81 = 4/81
et
F(2) = P(X ≤ 2) = p(0) + p(1) + p(2)
2
= 4/81 + 5/81 = 9/81
F(3) = P(X ≤ 3) = p(0) + p(1) + p(2) + p(3)
3
= 9/81 + 7/81 = 16/81
F(4) = P(X ≤ 4) = p(0) + p(1) + …. + p(4)
4
= 16/81 + 9/81 = 25/81

Downloaded From : www.EasyEngineering.net


Downloaded From : www.EasyEngineering.net

MA6451 PROBABILITY AND RANDOM PROCESSES

F(5) = P(X ≤ 5) = p(0) + p(1) + ….. + p(4) + p(5)


5
= 2/81 + 11/81 = 36/81
F(6) = P(X ≤ 6) = p(0) + p(1) + ….. + p(6)
6
= 36/81 + 13/81 = 49/81
F(7) = P(X ≤ 7) = p(0) + p(1) + …. + p(6) + p(7)
7
= 49/81 + 15/81 = 64/81
F(8) = P(X ≤ 8) = p(0) + p(1) + ….. + p(6) + p(7) +
8 p(8)
= 64/81 + 17/81 = 81/81 = 1

ww
1.3 CONTINUOUS RANDOM VARIABLE
Def : A R.V.’X’ which takes all possible values in a given internal is called a continuous random
variable.

w.E
Example : Age, height, weight are continuous R.V.’s.

asy
1.3.1 PROBABILITY DENSITY FUNCTION
Consider a continuous R.V. ‘X’ specified on a certain interval (a, b) (which can also be a infinite
interval (-∞, ∞)).

P(x < X < x + ∆x)En


If there is a function y = f(x) such that
= f (x)
lim
∆x → 0 ∆x
gin
Then this function f(x) is termed as the probability density function (or) simply density function
of the R.V. ‘X’.
ee rin
It is also called the frequency function, distribution density or the probability density
function.

Remark g.n
The curve y = f(x) is called the probability curve of the distribution curve.

P(a < x < b)


b
= ∫ f (x) dx
et
If f(x) is p.d.f of the R.V.X then the probability that a value of the R.V. X will fall in
some interval (a, b) is equal to the definite integral of the function f(x) a to b.

a (or)
b
P(a ≤ X ≤ b) = ∫ f (x) dx
a

1.3.2 PROPERTIES OF P.D.F


The p.d.f f(x) of a R.V.X has the following properties

(i) f(x) ≥ 0, -∞ < x < ∞ (ii) ∫ f (x) dx = 1
−∞

Remark
5

Downloaded From : www.EasyEngineering.net


Downloaded From : www.EasyEngineering.net

MA6451 PROBABILITY AND RANDOM PROCESSES

1. In the case of discrete R.V. the probability at a point say at x = c is not zero. But in the case
of a continuous R.V.X the probability at a point is always zero.

P ( X = c ) = ∫ f (x) dx = [ x ]c = C − C = 0
C

−∞

2. If x is a continuous R.V. then we have p(a ≤ X ≤ b) = p(a ≤ X < b)


= p(a < X V b)

IMPORTANT DEFINITIONS INTERMS OF P.D.F


If f(x) is the p.d.f of a random variable ‘X’ which is defined in the interval (a, b) then

ww
b
i Arithmetic mean ∫ x f (x) dx
a
b 1

w.E ii Harmonic mean ∫


a x
b
f (x) dx

∫ log x f (x) dx
iii
asy
Geometric mean ‘G’ log G
a
b
∫ x f (x) dx
r
iv
En
Moments about origin
a
b
v
gin
Moments about any point A ∫ (x − A) f (x) dx
a
r

ee
b
∫ (x − mean) f (x) dx
r
vi Moment about mean µ r

rin
a
b
∫ (x − mean) f (x) dx
2
vii Variance µ 2

viii Mean deviation about the mean is M.D.


a
b
g.n
∫ | x − mean | f (x) dx

1.3.3 Mathematical Expectations


a

et
Def :Let ‘X’ be a continuous random variable with probability density function f(x).
Then the mathematical expectation of ‘X’ is denoted by E(X) and is given by

E(X) = ∫ x f (x) dx
−∞

It is denoted by

µ 'r =
∫ x f (x) dx
r

−∞

Thus

Downloaded From : www.EasyEngineering.net


Downloaded From : www.EasyEngineering.net

MA6451 PROBABILITY AND RANDOM PROCESSES

µ1' =
E(X) (µ1' about origin)
µ '2 =
E(X 2 ) (µ '2 about origin)
∴  Mean =  X =µ1' =E(X)
And
Variance = µ '2 −µ '2 2
Variance= E(X 2 ) − [E(X)]2 (a)
th
* r moment (abut mean)
Now

E {X − E ( X )}
r
= ∫ {x − E(X)}r f (x) dx

ww =
−∞

∫ {x − X} f (x) dx
r

w.E
−∞

Thus

µr = ∫ {x − X} f (x) dx
r

Where µr = asy −∞

E[X − E(X) ] r
(b)

En
This gives the rth moment about mean and it is denoted by µ r
Put r = 1 in (B) we get
µr =

∫ {x − X}f (x) dx gin

−∞


= ∫ x f (x) dx − ∫ x f (x) dx
ee rin
=
−∞

X − X ∫ f (x) dx

−∞

 ∞ f (x) dx =
 −∞∫ 1 g.n
=
µ1
X−X
=0
−∞ 
et
Put r = 2 in (B), we get

µ2 = ∫ (x − X) f (x) dx
2

−∞

Variance =
µ2 =E[X − E(X)]2
Which gives the variance interms of expectations.
Note
Let g(x) = K (Constant), then

Downloaded From : www.EasyEngineering.net


Downloaded From : www.EasyEngineering.net

MA6451 PROBABILITY AND RANDOM PROCESSES


E g (=
X )     E (=
K) ∫ K f (x) dx
−∞

 ∞ f (x) dx =1
= K ∫ f (x) dx  −∞∫ 
−∞

= K.1 = K
Thus E(K) = K ⇒ E[a constant] = constant.
1.3.4 EXPECTATIONS (Discrete R.V.’s)
Let ‘X’ be a discrete random variable with P.M.F p(x)
Then
E(X) = ∑ x p(x)

ww
x
For discrete random variables ‘X’
E(X r ) = ∑ x p(x)
r

w.E
If we denote
= r
µ 'r
x (by def)

E(X
Then
)
asy
µ='
r =
E[X ] r

En
∑ x p(x)
r

Put r = 1, we get
Mean µ 'r = ∑ x p(x) gin
Put r = 2, we get
µ '2= =
E[X 2
]
ee
∑ x p(x)
2
rin

µ 2 = µ '2 −µ1' 2 = E(X 2 ) − {E(X)}
2
x

g.n
The rth moment about mean
µ 'r = E[{X − E(X)}r ] et
=∑ (x − X) p(x),
r
E(X) =
X
x
Put r = 2, we get
Variance = µ= 2 ∑ (x − X) p(x)
2

1.3.5 ADDITION THEOREM (EXPECTATION)


Theorem 1
If X and Y are two continuous random variable with pdf fx (x) and fy (y) then
E(X+Y) = E(X) + E(Y)

Downloaded From : www.EasyEngineering.net


Downloaded From : www.EasyEngineering.net

MA6451 PROBABILITY AND RANDOM PROCESSES

1.3.6 MULTIPLICATION THEOREM OF EXPECTATION


Theorem 2
If X and Y are independent random variables,
Then E(XY) = E(X) . E(Y)
Note :
If X 1 , X 2 , ……, X n are ‘n’ independent random variables, then
E[X 1 , X 2 , ……, X n] = E(X 1 ), E(X 2 ), ……, E(X n )

Theorem 3
If ‘X’ is a random variable with pdf f(x) and ‘a’ is a constant, then
(i) E[a G(x)] = a E[G(x)]
(ii) E[G(x)+a] = E[G(x)+a]

ww
Where G(X) is a function of ‘X’ which is also a random variable.

Theorem 4

w.EIf ‘X’ is a random variable with p.d.f. f(x) and ‘a’ and ‘b’ are constants, then
E[ax + b] = a E(X) + b

Cor 1:
asy
If we take a = 1 and b = –E(X) = – X , then we get

En
E(X- X ) = E(X) – E(X) = 0
Note
1
E  ≠
1 gin
 X  E(X)
E[log (x)] ≠ log E(X)
E(X2) ≠ [E(X)]2
ee
1.3.7 EXPECTATION OF A LINEAR COMBINATION OF RANDOM VARIABLES rin
g.n
Let X 1 , X 2 , ……, X n be any ‘n’ random variable and if a 1 , a 2 , ……, a n are constants, then
E[a 1 X 1 + a 2 X 2 + ……+ a nX n ] = a 1 E(X 1 ) + a 2 E(X 2 )+ ……+ a nE(X n )
Result
If X is a random variable, then
Var (aX + b) = a2Var(X) ‘a’ and ‘b’ are constants.
Covariance :
et
If X and Y are random variables, then covariance between them is defined as
Cov(X, Y) = E{[X - E(X)] [Y - E(Y)]}
= E{XY - XE(Y) – E(X)Y + E(X)E(Y)}
Cov(X, Y) = E(XY) – E(X) . E(Y) (A)
If X and Y are independent, then
E(XY) = E(X) E(Y)
Sub (B) in (A), we get
Cov (X, Y) = 0
∴ If X and Y are independent, then
9

Downloaded From : www.EasyEngineering.net


Downloaded From : www.EasyEngineering.net

MA6451 PROBABILITY AND RANDOM PROCESSES

Cov (X, Y) = 0
Note
(i) Cov(aX, bY) = ab Cov(X, Y)
(ii) Cov(X+a, Y+b) = Cov(X, Y)
(iii) Cov(aX+b, cY+d) = ac Cov(X, Y)
(iv) Var (X 1 + X 2 ) = Var(X 1 ) + Var(X 2 ) + 2 Cov(X 1 , X 2 )
If X 1 , X 2 are independent
Var (X 1 + X 2 ) = Var(X 1 ) + Var(X 2 )
EXPECTATION TABLE
Discrete R.V’s Continuous R.V’s

1. E(X) = ∑x p(x) 1. E(X) = ∫ x f (x) dx

ww
2. E(X ) =µ =∑ x p(x)
r '
r
x
r
−∞

2. E(X r ) =µ 'r = ∫ x r f (x) dx

w.E
−∞

3. Mean = µ 'r =∑ x p(x) 3. Mean = µ 'r = ∫ x f (x) dx
−∞

4. µ '2 =∑ x 2 p(x)
asy ∞
4. µ '2 = ∫ x 2 f (x) dx
−∞

5. Variance = µ '2 −µ1' 2 = E(X2) – {E(X)}2


En 5. Variance = µ '2 −µ1' 2 = E(X2) – {E(X)}2

gin
SOLVED PROBLEMS ON DISCRETE R.V’S
Example :1
ee rin
When die is thrown, ‘X’ denotes the number that turns up. Find E(X), E(X2) and Var (X).
Solution
Let ‘X’ be the R.V. denoting the number that turns up in a die.
‘X’ takes values 1, 2, 3, 4, 5, 6 and with probability 1/6 for each g.n
X=x

p(x)
1
1/6
2
1/6
3
1/6
4
1/6
5
1/6
6
1/6 et
p(x1 ) p(x2 ) p(x3 ) p(x4 ) p(x5 ) p(x6 )

Now
6
E(X) = ∑ x i p(x i )
i =1
= x1 p(x 1 ) + x2 p(x2 ) + x3 p(x3 ) + x 4 p(x 4 ) + x5 p(x 5 ) + x6 p(x6 )
= 1 x (1/6) + 1 x (1/6) + 3 x (1/6) + 4 x (1/6) + 5 x (1/6) + 6 x (1/6)
= 21/6 = 7/2 (1)

10

Downloaded From : www.EasyEngineering.net


Downloaded From : www.EasyEngineering.net

MA6451 PROBABILITY AND RANDOM PROCESSES

6
E(X) = ∑ x i p(x p )
i =1
= x1 2p(x1 )+x 2 2p(x 2 )+x3 2p(x3 )+x 4 2p(x4 )+x 5 2p(x 5 )+x 6 p(x6 )
= 1(1/6) + 4(1/6) + 9(1/6) + 16(1/6) + 25(1/6) + 36(1/6)
1 + 4 + 9 + 16 + 25 + 36 91
= = (2)
6 6
Variance (X) = Var (X) = E(X2) – [E(X)]2
2
91  7  91 49 35
= –  = − =
6 2 6 4 12
Example :2
Find the value of (i) C (ii) mean of the following distribution given

ww C(x − x 2 ),
f (x) = 
0
0 < x <1
otherwise

w.E
Solution

Given f (x) = 
C(x − x 2 ), 0 < x <1
(1)


∫ f (x) dx =1
0
asy otherwise

−∞
1
∫ C(x − x ) dx =
2 En
0

 x 2 x3 
1
1 gin
[using (1)] [∴ 0<x<1]

C −  =
2
1 1
C −  =
3 0

1
1
ee rin
 2 3
3 − 2
=1
g.n
C

C
6
 6 
=1 C=6 (2)
et
Sub (2) in (1), f(x) = 6(x – x2), 0< x < 1 (3)

Mean = E(x) = ∫ x f (x) dx
−∞
1
= ∫ x 6(x − x 2 ) dx [from (3)] [∴ 0 < x < 1]
0
1
= ∫ (6x 2 − x 3 ) dx
0

11

Downloaded From : www.EasyEngineering.net


Downloaded From : www.EasyEngineering.net

MA6451 PROBABILITY AND RANDOM PROCESSES

1
 6x 3 6x 4 
=  −
 3 4  0
∴ Mean = ½
Mean C
½ 6

1.4 CONTINUOUS DISTRIBUTION FUNCTION


Def :
If f(x) is a p.d.f. of a continuous random variable ‘X’, then the function

F X (x) = F(x) = P(X ≤ x) = ∫ f (x) dx, − ∞ < x < ∞

ww −∞
is called the distribution function or cumulative distribution function of the random
variable.

(i)
(ii)
w.E
* PROPERTIES OF CDF OF A R.V. ‘X’
0 ≤ F(x) ≤ 1, - ∞ < x < ∞
Lt F(x) = 0 , Lt F(x) = 1

(iii)
x →−∞
b
P(a ≤ X ≤ b) = ∫ f (x)=
dx asy
x →−∞

F(b) − F(a)

F'(x) =
dF(x)
a

En f(x) ≥ 0
(iv)
(v)
dx
P(X = xi ) = F(x i ) – F(xi – 1)
=
gin
Example :1.4.1

6x(1 − x),
f (x) = 
0 < x <1
ee
Given the p.d.f. of a continuous random variable ‘X’ follows

, find c.d.f. for ‘X’ rin


Solution
0 otherwise
g.n
Given f (x) = 
6x(1 − x),
0
x
0 < x <1
otherwise et
=
The c.d.f is F(x) ∫ f (x) dx , − ∞ < x < ∞
−∞
(i) When x < 0, then
x
F(x) = ∫ f (x) dx
−∞
x
= ∫ 0 dx =0
−∞
(ii) When 0< x < 1, then

12

Downloaded From : www.EasyEngineering.net


Downloaded From : www.EasyEngineering.net

MA6451 PROBABILITY AND RANDOM PROCESSES

x
F(x) = ∫ f (x) dx
−∞
0 x
= ∫ f (x) dx + ∫ f (x) dx
−∞ 0
x
x  x 2 x3 
x
= 0 + ∫ 6x(1 − x) dx = 6 ∫ x(1 − x) dx = 6  − 
0 0 2 3 0
= 3x 2 − 2x 3
(iii) When x > 1, then
x
F(x) = ∫ f (x) dx
−∞

ww
0 1 x
= ∫ 0dx + ∫ 6x(1 − x) dx + ∫ 0 dx
−∞ 0 0
1

w.E = 6 ∫ (x − x 2 ) dx
Using (1), (2) & (3) we get
0
=1

F(x)
0,

= 3x 2 − 2x 3 , asy x<0
0 < x <1
1,
 En x >1

Example:1.4.2 gin
(i) If f (x) = 
e − x ,
0,
x≥0
x<0 ee
defined as follows a density function ?

rin
(ii) If so determine the probability that the variate having this density will fall in the interval (1,
2).
Solution g.n
Given

(a) In (0, ∞), e-x is +ve


e − x ,
f (x) = 
0,
x≥0
x<0 et
∴f(x) ≥ 0 in (0, ∞)

∞ 0 ∞
(b) ∫ f (x) dx = ∫ f (x) dx + ∫ f (x) dx
−∞ −∞ 0
0 ∞
= ∫ 0dx + ∫ e − x dx
−∞ 0

=  −e − x  − e −∞ + 1
=
0

13

Downloaded From : www.EasyEngineering.net


Downloaded From : www.EasyEngineering.net

MA6451 PROBABILITY AND RANDOM PROCESSES

=1
Hence f(x) is a p.d.f
(ii) We know that
b
P(a ≤ X ≤ b) = ∫ f (x) dx
a
2 2
P(1 ≤ X ≤ 2) = ∫ f (x) dx = ∫ e − x dx = [−e − x ]2+1
1 1
2
= ∫ e − x dx = [−e − x ]2+1
1
= -e-2 + e-1 = -0.135 + 0.368 = 0.233

Example:1.4..3

ww A probability curve y = f(x) has a range from 0 to ∞. If f(x) = e-x, find the mean and
variance and the third moment about mean.

w.E
Solution
Mean

= ∫ x f (x) dx
0

= ∫ x e − x dx
0 asy =  x[−e − x ] − [e − x ]

Mean = 1

Variance µ 2= ∫ (x − Mean) 2 f (x) dx En

0

= ∫ (x − 1) 2 e − x dx
gin
µ 2 =1
Third moment about mean
0
ee rin
b
µ3= ∫ (x − Mean)3 f (x) dx g.n
et
a
Here a = 0, b = ∞
b
µ3 = ∫ (x − 1)3 e − x dx
a

{ }

= (x − 1)3 ( −e − x ) − 3(x − 1) 2 (e − x ) + 6(x − 1)( −e − x ) − 6(e − x )
0
= -1 + 3 -6 + 6 = 2
µ3 = 2
1.5 MOMENT GENERATING FUNCTION
Def : The moment generating function (MGF) of a random variable ‘X’ (about origin) whose
probability function f(x) is given by
M X(t) = E[etX]

14

Downloaded From : www.EasyEngineering.net


Downloaded From : www.EasyEngineering.net

MA6451 PROBABILITY AND RANDOM PROCESSES

 ∞ e tx f (x)dx, for a continuous probably function


 ∫
=  x = −∞

 ∑ e tx p(x), for a discrete probably function
 x = −∞
Where t is real parameter and the integration or summation being extended to the entire range of
x.
Example :1.5.1
∞ tr '
Prove that the rth moment of the R.V. ‘X’ about origin is M=
X (t) ∫ µr
r = 0 r!
Proof
WKT M X (t) = E(etX)
 tX (tX) 2 (tX)3 
ww = E 1 +
 1!
+
2!
+
3!
+ .... + +
(tX) r
r!
+ ....

w.E
2 r
t t
= E[1] + t E(X) + E(X 2 ) + ..... + E(X r ) + ........
2! r!
2 3 r
M X (t)

[using µ 'r =E(X r ) ]


asy t
2!
t
3!
t
= 1 + t µ1' + µ '2 + µ3' + ..... + µ 'r + ........
r!

En tr
Thus rth moment = coefficient of

Note gin
r!

ee
1. The above results gives MGF interms of moments.
2. Since M X (t) generates moments, it is known as moment generating function.
Example:1.5.2
rin
Find µ1' and µ '2 from M X (t)
Proof g.n
WKT M X (t) = ∑ µ r

M X (t)
t
tr '
r = 0 r!

t2

tr
=µ '0 + µ1' + µ '2 + ..... + µ 'r
et
(A)
1! 2! r!
Differenting (A) W.R.T ‘t’, we get
2t ' t 3 '
'
M X (t) =µ + µ 2 + µ3 + .....
'
1 (B)
2! 3!
Put t = 0 in (B), we get
M X ' (0) =µ1' =Mean
d 
Mean = M1' (0) (or)  dt (M X (t)) 
t =0
15

Downloaded From : www.EasyEngineering.net


Downloaded From : www.EasyEngineering.net

MA6451 PROBABILITY AND RANDOM PROCESSES

M X" (t) =µ '2 + t µ3' + .......


Put t = 0 in (B)
 d2 
M X (0) = µ
" '
2 (or)  dt 2 (M X (t)) 
  t =0
 dr 
In general µ = r (M X (t)) 
'
r
 dt  t =0
Example :1.5.3
Obtain the MGF of X about the point X = a.
Proof
The moment generating function of X about the point X = a is M X (t) = E[e t (X −a ) ]

ww  t2 tr
= E 1 + t(X − a) + (X − a) 2 + .... + (X − a) r + ....
 2! r!

 w.E
 Formula
2
e x =1 + x + x + ...


 1! 2! 
asy t2 tr
= E (1) + E[t(X − a)] + E[ (X − a) 2 ] + .... + E[ (X − a) r ] + ....
2 En
2!
r
r!
t
gin t
=1 + t E(X − a) + E(X − a) 2 + .... + E(X − a) r + ....
2! r!

ee
2 r
t t
= 1 + t µ1' + µ '2 + .... + µ 'r + .... Where = µ 'r E[(X − a) r ]
2!
2
t '
r!
tr '
[ M X (t)]x =a = 1 + tµ1 + µ 2 + ..... + µr + .....
'
rin
Result:
2! r!
g.n
M CX (t) = E[e tcx ]
M X (t) = E[e ]
From (1) & (2) we get
ctx
(1)
(2) et
M CX (t) = M X (ct)
Example :1.5.4
If X 1 , X2, ….., Xn are independent variables, then prove that
t (X1 + X 2 +....+ X n )
M X1 + X2 +....+ Xn (t) = E[e ]
= E[e tX1 .e tX 2 .....e tX n ]
= E(e tX1 ).E(e tX 2 ).....E(e tX n )
[∴ X 1 , X 2 , ….., X n are independent]

16

Downloaded From : www.EasyEngineering.net


Downloaded From : www.EasyEngineering.net

MA6451 PROBABILITY AND RANDOM PROCESSES

= M X1 (t).M X 2 (t)..........M X n (t)

Example:1.5.5
− at t
X−a  
Prove that if ∪ = , then M ∪ (t) = e h .M X  h  , where a, h are constants.
h
Proof
By definition
M ∪ (t) = E e tu    M X (t) = E[e tx ]
 t  Xh−a  
= E e   
 

ww  tXn − tan 
= E e



w.E tX
= E[ e h ] E[ e
− ta
h ]

asy
− ta tX
=e h E[ e ] h [by def]
− ta t

En
 
h h
=e . MX
− at
t X−a
∴ M ∪ (t) = e h

h gin
.M X   , where ∪ =
h
and M X(t) is the MGF about origin.

Example:1.5.6
Find the MGF for the distribution where
2
ee rin
3

1
at x = 1
g.n
=f (x) =

0

3
at x 2

otherwise
et

Solution
2
Given f (1) =
3
1
f (2) =
3
f(3) = f(4) = …… = 0
MGF of a R.V. ‘X’ is given by

17

Downloaded From : www.EasyEngineering.net


Downloaded From : www.EasyEngineering.net

MA6451 PROBABILITY AND RANDOM PROCESSES

M X (t) = E[e tx ]

= ∑ e tx f (x)
x =0
= e0 f(0) + et f(1) + e2t f(2) + …….
= 0 +et f(2/3) + e2t f(1/3) + 0
= 2/3et + 1/3e2t
et
∴ MGF is M=
X (t) [2 + e t ]
3
1.6 Discrete Distributions
The important discrete distribution of a random variable ‘X’ are
1. Binomial Distribution
2. Poisson Distribution

ww 3. Geometric Distribution
1.6.1 BINOMIAL DISTRIBUTION

by
w.E
Def : A random variable X is said to follow binomial distribution if its probability law is given

P(x) = p(X = x successes) = nC x p x qn-x


Where x = 0, 1, 2, ……., n, p+q = 1
Note
Assumptions in Binomial distribution asy
En
i) There are only two possible outcomes for each trail (success or failure).
ii) The probability of a success is the same for each trail.

gin
iii) There are ‘n’ trails, where ‘n’ is a constant.
iv) The ‘n’ trails are independent.

Example :1.6.1
ee rin
Find the Moment Generating Function (MGF) of a binomial distribution about origin.
Solution
n
M X (t) = ∑ e tx p(x)
g.n
WKT
x =0
Let ‘X’ be a random variable which follows binomial distribution then MGF about origin is
given by
n
et
E[e=
tX
] =
M X (t) ∑ e p(x)
tx

x =0
n
= x n −x
∑ e nC x p q
tx
 p(x) = nC x p x q n − x 
x =0
n
n−x
= ∑ (e ) p nC x q
tx x

x =0
n
n−x
= ∑ (pe ) nC x q
t x

x =0

18

Downloaded From : www.EasyEngineering.net


Downloaded From : www.EasyEngineering.net

MA6451 PROBABILITY AND RANDOM PROCESSES

∴ M X (t) =
(q + pe t ) n
Example:1.6.2
Find the mean and variance of binomial distribution.
Solution
M X (t) = (q + pe t ) n
∴ M 'X (t) n(q + pe t ) n −1.pe t
=
Put t = 0, we get
M 'X (0) = n(q + p) n −1.p
Mean = E(X) = np [ (q + p) =
1]  Mean M 'X (0) 
M"X (t) = np (q + pe t ) n −1.e t + e t (n − 1)(q + pe t ) n − 2 .pe t 

ww
Put t = 0, we get
M"X (t) = np (q + p) n −1 + (n − 1)(q + p) n − 2 .p 

w.E = np [1 + (n − 1)p ]
= np + n 2 p 2 − np 2

asy
= n 2 p 2 + np(1 − p)
[ 1 − p =q]
M"X (0)
M"X (0)
=
= =
E(X 2
En
n 2 p 2 + npq
) n 2 p 2 + npq
Var ( X ) = gin
E(X 2 ) − [E(X)]2= n 2 / p 2 + npq − n 2 / p 2= npq
Var (X) = npq
S.D = npq
Example :1.6.3
ee rin
(np).
Find the Moment Generating Function (MGF) of a binomial distribution about mean
g.n
et
Solution
Wkt the MGF of a random variable X about any point ‘a’ is
M x(t) (about X = a) = E[et(X-a)]
Here ‘a’ is mean of the binomial distribution
M X(t) (about X = np) = E[et(X-np)]
= E[etX . e-tnp)]
= e-tnp . [-[e-tX)]]
= e-tnp . (q+pet)-n
= (e-tp)n. (q + pet)n
∴ MGF about mean = (e-tp)n. (q + pet)n

Example :1.6.4
Additive property of binomial distribution.
Solution
19

Downloaded From : www.EasyEngineering.net


Downloaded From : www.EasyEngineering.net

MA6451 PROBABILITY AND RANDOM PROCESSES

The sum of two binomial variants is not a binomial variate.


Let X and Y be two independent binomial variates with parameter
(n1 , p 1 ) and (n 2 , p 2 ) respectively.
Then
( t )  (q + p 1e t ) , ( t )  (q + p 2e t )
n1 n2
MX = 1 MY= 2

∴ M X+Y ( 
t) =
M X ( t ) .M Y ( t ) [ X & Y are independent R.V.’s]

( ) . (q + p 2e t )
n1 n2
= q1 + p1e t 2

( ) . Hence by uniqueness theorem of


n
RHS cannot be expressed in the form q + pe t
MGF X+Y is not a binomial variate. Hence in general, the sum of two binomial variates is not a
binomial variate.

ww ( ) ( )
Example :1.6.5
=
w.(E )
If M X (t) =
n1
q+pe t , M Y (t)

M X + Y (t) = q+pe t
n1 + n 2
q+pe t
n2
, then

asy
Problems on Binomial Distribution
1. Check whether the following data follow a binomial distribution or not. Mean = 3; variance =
4.
Solution En
Given
gin Mean np = 3
Variance npr = 4 (2)
(1)

(2)
(1)
4
np

ee
=
npq 4
1
3

rin
⇒ q = = 1 which is > 1.
3 3
Since q > 1 which is not possible (0 < q < 1). The given data not follow binomial distribution. g.n
Example :1.6.5
The mean and SD of a binomial distribution are 5 and 2, determine the distribution.
Solution
Given Mean = np = 5 (1)
et
SD = npq = 2 (2)
(2) np 4 4
⇒ =⇒ q=
(1) npq 5 5
4 1 1
∴ p =1 − = ⇒ p=
5 5 5
Sub (3) in (1) we get
n x 1/5 = 5

20

Downloaded From : www.EasyEngineering.net


Downloaded From : www.EasyEngineering.net

MA6451 PROBABILITY AND RANDOM PROCESSES

n = 25
∴ The binomial distribution is
P(X = x) = p(x) = nC x px qn-x
= 25C x(1/5)x(4/5)n-x, x = 0, 1, 2, ….., 25

1.7 Passion Distribution


Def :
A random variable X is said to follow if its probability law is given by
e −λ λ x
P(X = x) = p(x) = , x = 0, 1, 2, ….., ∞
x!

Poisson distribution is a limiting case of binomial distribution under the following

ww
conditions or assumptions.
1. The number of trails ‘n’ should e infinitely large i.e. n→∞.
2. The probability of successes ‘p’ for each trail is infinitely small.

w.E
3. np = λ , should be finite where λ is a constant.
* To find MGF
M X(t) = E(etx)

asy
= ∑ e tx p(x)
x =0

= ∑ e tx  En
 λ x eλ 

= ∑
x =0

gin
 x! 
∞ e −λ (λe t ) x

x =0

= e −λ ∑
x =0
x!
∞ (λe t ) x

x!
ee rin

= e −λ 1 + λe t +

(λe t ) 2
2!

+ ......

g.n
Hence
= e −λ eλe = eλ (e −1)
t

M X(t) = eλ (e −1)
t
t

et
* To find Mean and Variance
= eλ (e −1)
t
WKT M X (t)
= eλ (e −1) .e t
t
∴ M X '(t)
M X '(0) = e −λ . λ

=µ1' E(X)
= ∑ x.p(x)
x =0

21

Downloaded From : www.EasyEngineering.net


Downloaded From : www.EasyEngineering.net

MA6451 PROBABILITY AND RANDOM PROCESSES

e −λ λ x∞ ∞ x. e −λ λλ x −1
= ∑ x. = ∑
= x 0= x! x 0 x!
∞ x.λ x −1
= 0 + e −λ . λ ∑
x =1 x!

∞ λ x 1
= λ e −λ . ∑
x =1 (x − 1)!

 λ2 
= λ e −λ 1 + λ + + .....
 2! 
= λ e −λ .eλ
Mean = λ

ww
µ = E[X ] = ∑ x .p(x) = ∑ x .
'
2
=
2

x 0=

2

x 0
e −λ λ x

x!
2

w.E ∞
= ∑ {x(x − 1) + x}.
x =0
−λ
e λ
x!
x

= ∑
=
∞ x(x − 1)e λ

x 0= x! asy
−λ x
+∑
∞ x.e −λ λ x

x 0 x!
= e −λ λ 2 ∑

En
λ x −2

x = 0 (x − 2)(x − 3)....1

= e −λ λ 2 ∑
∞ λ x −2
+λgin
−λ 2 
= e λ 1 + +
x = 0 (x − 2)!

 1! 2!
λ λ2 ee

+ .... + λ
 rin
= λ +λ
2

Variance µ 2 = E(X 2 ) − [E(X)]2 = λ2 + λ − λ2 = λ


g.n
Variance = λ
Hence Mean = Variance =λ
Note : * sum of independent Poisson Vairates is also Poisson variate.
et
PROBLEMS ON POISSON DISTRIBUTION
Example:1.7.1
3 1
If x is a Poisson variate such that P(X=1) = and P(X=2) = , find the P(X=0) and P(X=3).
10 5
Solution
e −λ λ x
P(X = x) =
x!
22

Downloaded From : www.EasyEngineering.net


Downloaded From : www.EasyEngineering.net

MA6451 PROBABILITY AND RANDOM PROCESSES

3
∴ P(X=1) = e −λ λ = (Given)
10
3
= λe −λ = (1)
10
e −λ λ 2 1
P(X=2) = = (Given)
2! 5
e −λ λ 2 1
= (2)
2! 5
3
(1) ⇒ e −λ λ = (3)
10

ww (2) ⇒ e −λ λ 2 =

(3) 1 3
2
5
(4)

w.E (4)

λ=
λ 4
=

4
3
asy
e −λ λ 0
∴ P(X=0) =
En
0!
−λ 3
= e −4/3

e −4/3 (4 / 3)3
P(X=3) =
e λ
3!gin =
3!

Example :1.7.2
If X is a Poisson variable
P(X = 2) = 9 P(X = 4) + 90 P(X=6)
ee rin
Find
Solution
(i) Mean if X (ii) Variance of X
g.n
P(X=x) =
Given
e −λ λ x
x!
, x = 0,1, 2,.....
P(X = 2) = 9 P(X = 4) + 90 P(X=6)
et
e −λ λ 2 e −λ λ 4 e −λ λ 6
=9 + 90
2! 4! 6!
1 9λ 90λ 2 4
= +
2 4! 6!
1 3λ 2
λ 4
= +
2 8 8
3λ 2
λ4
1= +
4 4
23

Downloaded From : www.EasyEngineering.net


Downloaded From : www.EasyEngineering.net

MA6451 PROBABILITY AND RANDOM PROCESSES

λ 4 + 3λ 2 − 4 =0
λ2 = 1 or λ2 = -4
λ =±1 or λ = ± 2i
∴ Mean = λ = 1, Variance = λ = 1
∴ Standard Deviation = 1

1.7.3 Derive probability mass function of Poisson distribution as a limiting case of Binomial
distribution
Solution
We know that the Binomial distribution is
P(X=x) = nC x pxqn-x
n!
p x (1 − p) n − x
ww =
(n − x)! x!
1.2.3.......(n − x)(n − x + 1)......np n (1 − p) n

w.E =
1.2.3.....(n − x) x! (1 − p) x
1.2.3.......(n − x)(n − x + 1)......n  p 
x

=
asy
1.2.3.....(n − x) x!
  (1 − p)
1− p 
n

En
n(n − 1)(n − 2)......(n − x + 1) λ x  λ
n
1
= x 
1− 
x! n  λ  n
x

gin 1 − 
 n
−x

ee
n(n − 1)(n − 2)......(n − x + 1)  λ   λ 
n

= x 1 −  1 − 
 n  n
 1  2 
x!
  x − 1 
11 − 1 −  ...... 1 −  
rin
P(X=x) =
 n  n 
x!
  n  x  λ 
λ 1 − 
 n
n−x

g.n
=
λ  1  2 
x
  x − 1   λ 
11 − 1 −  ...... 1 − 
x!  n  n 
  1 − 
  n   n 
n−x
et
When n→∞
λx   1  2    x − 1   λ  
n−x

P(X=x) = lt 1 − 1 − 1 −  ...... 1 −    1 −  


x! n →∞   n  n    n    n  
λx  1  2  x −1 
= lt 1 −  lt 1 −  ...... lt 1 −  
x! n →∞  n  n →∞  n  n →∞
 n 
We know that

24

Downloaded From : www.EasyEngineering.net


Downloaded From : www.EasyEngineering.net

MA6451 PROBABILITY AND RANDOM PROCESSES

n −x
 λ
lt 1 −  e −λ
=
n →∞
 n
 1  2   x −1  
and lt 1 − = lt 1 −  .....= lt 1 −   = 1
n →∞
 n n →∞
 n n →∞
  n 
λ x −λ
∴ P(X=x) = = e , x 0,1, 2,...... ∞
x!

1.8 GEOMETRIC DISTRIBUTION


Def: A discrete random variable ‘X’ is said to follow geometric distribution, if it assumes only
non-negative values and its probability mass function is given by
P(X=x) = p(x) = qx-1 ; x = 1, 2, ……., 0 < p < 1, Where q = 1-p

ww
Example:1.8.1
To find MGF
= E[etx]
w.E M X(t)
= ∑ e tx p(x)

asy
= ∑ e tx q x −1p
x =1

En
= ∑ e tx q x q −1p
x =1

= ∑ e tx q x p / q
x =1 gin

= p / q ∑ e tx q x
x =1

= p / q ∑ (e t q) x
ee rin
x =1

= p / q (e t q)1 + (e t q) 2 + (e t q)3 + .... g.n


Let x = etq = p / q  x + x 2 + x 3 + ....

=
p
x 1 + x + x 2 + ....
p
= (1 − x) −1
et
q q
p
= qe t 1 − qe t  = pe t [1 − qe t ]−1
q
pe t
∴M X (t) =
1 − qe t

* To find the Mean & Variance

25

Downloaded From : www.EasyEngineering.net


Downloaded From : www.EasyEngineering.net

MA6451 PROBABILITY AND RANDOM PROCESSES

(1 − qe t )pe t − pe t (−qe t ) pe t
M 'X ( t )    = =
(1 − qe t ) 2 (1 − qe t ) 2
∴ E(X) = M 'X ( 0 )   = 1/p
∴ Mean = 1/p
d  pe t 
µ"X (t) =
dt  (1 − qe t ) 2 
Variance

(1 − qe t ) 2 pe t − pe t 2(1 − qe t )(−qe t )
=
(1 − qe t ) 4
(1 − qe t ) 2 pe t + 2pe t qe t (1 − qe t )
=
(1 − qe t ) 4

ww 1+ q
M"X (0) = 2
p

w.E Var (X) = E(X2) – [E(X)]2 =


(1 + q) 1
p 2
− 2 ⇒ 2
p p
q

Var (X) = 2
p
q
asy
Note:
Another form of geometric distribution En
P[X=x] = qxp ; x = 0, 1, 2, ….

M X (t) =
p gin
Mean = q/p,
(1 − qe t )
Variance = q/p2 ee rin
Example:1.8.2
If the MGF of X is (5-4et)-1, find the distribution of X and P(X=5) g.n
Solution
Let the geometric distribution be
P(X = x) = qxp, x = 0, 1, 2, …..
The MGF of geometric distribution is given by
et
p
(1)
1 − qe t
1
 4  −1
Here M X(t) = (5 - 4e ) ⇒ 5 1 − e t 
t -1
(2)
 5 
4 1
Company (1) & (2) we get q= ; p=
5 5
∴ P(X = x) = pqx, x = 0, 1, 2, 3, …….

26

Downloaded From : www.EasyEngineering.net


Downloaded From : www.EasyEngineering.net

MA6451 PROBABILITY AND RANDOM PROCESSES

x
 1  4 
=   
 5  5 
5
 1  4  45
P(X = 5) =    = 6
 5  5  5
1.9 CONTINUOUS DISTRIBUTIONS
If ‘X’ is a continuous random variable then we have the following distribution
1. Uniform (Rectangular Distribution)
2. Exponential Distribution
3. Gamma Distribution
4. Normal Distribution
1. 9.1 Uniform Distribution (Rectangular Distribution)

ww
Def : A random variable X is set to follow uniform distribution if its
 1

f (x) =  b − a
, a<x<b

w.E
* To find MGF
0, otherwise

M X (t)

asy
= ∫ e tx f (x)dx
−∞
b
= ∫ e tx
1
b−a En
dx

=
1  e tx 
b − a  t  b
a

gin
a

=
1
(b − a)t
e bx − eat 
ee rin
∴ The MGF of uniform distribution is
e bt − eat g.n
M X (t) =
* To find Mean and Variance
(b − a)t


et
E(X) = ∫ x f (x)dx
−∞
b
 x2 
 
1 b 1 b  2 a
= ∫ bx
dx = ∫ x dx =
a b−a b−a a b−a
b −a
2 2
b+a a+b
= = =
2(b − a) 2 2

27

Downloaded From : www.EasyEngineering.net


Downloaded From : www.EasyEngineering.net

MA6451 PROBABILITY AND RANDOM PROCESSES

a+b
Mean µ1' =
2
Putting r = 2 in (A), we get
b x2 b
µ = ∫ x f (x)dx
'
= ∫ 2
dx
a b−a
2
a

a 2 + ab + b 2
=
3
∴ Variance = µ 2 − µ1' 2
'

b 2 + ab + b 2  b + a  (b − a) 2
2

= −  =
3  2  12

ww Variance =
(b − a) 2
12

w.E
PROBLEMS ON UNIFORM DISTRIBUTION
Example 1.9.1

(i) P(X>1) = 1/3 asy


If X is uniformly distributed over (-α,α), α< 0, find α so that

Solution
(ii) P(|X| < 1) = P(|X| > 1)

En
If X is uniformly distributed in (-α, α), then its p.d.f. is
 1

=  2α
−α < x < α gin
(i)
f (x)

P(X>1) = 1/3
0 otherwise ee rin
g.n
α
∫ f (x)dx = 1 / 3
1
α

1
1
1 2α
dx = 1 / 3

( x )1 = 1 / 3
α

1
( α − 1) =1 / 3
et
2α 2α
α=3
(ii) P(|X| < 1) = P(|X| > 1) = 1 - P(|X| < 1)
P(|X| < 1) + P(|X| < 1) = 1
2 P(|X| < 1) = 1
2 P(-1 < X < 1) = 1
1
2 ∫ f (x)dx = 1
1

28

Downloaded From : www.EasyEngineering.net


Downloaded From : www.EasyEngineering.net

MA6451 PROBABILITY AND RANDOM PROCESSES

1 1
2/ ∫ dx = 1

12
⇒α = 2

Note:
1. The distribution function F(x) is given by
0 −α < x < α
x − a

= 
F(x) a≤x≤b
 b − a
1 b<x<∞
2. The p.d.f. of a uniform variate ‘X’ in (-a, a) is given by

ww F(x)
1

=  2a
−a < x < a

w.E 0 otherwise

asy
1.10 THE EXPONENTIAL DISTRIBUTION
Def :A continuous random variable ‘X’ is said to follow an exponential distribution with
parameter λ>0 if its probability density function is given by

F(x)
λe
=
−λx
x>a
En
To find MGF
0 otherwise
gin
Solution
M X (t)

= ∫ e tx f (x)dx
−∞

ee ∞ rin
= ∫ e λe
0
tx −λx
dx =λ∫e
0
− ( λ− t )x
dx
g.n
et

 e − ( λ− t )x 
= λ 
 λ − t 0
λ λ
= e −∞ − e −0  =
−(λ − t) λ−t
λ
∴ MGF of x = , λ> t
λ−t

* To find Mean and Variance


We know that MGF is

29

Downloaded From : www.EasyEngineering.net


Downloaded From : www.EasyEngineering.net

MA6451 PROBABILITY AND RANDOM PROCESSES

−1
λ 1  t
M X(t)= = = 1 − 
λ − t 1− t  λ 
λ
2
t t tr
=+ 1 + 2 + ..... + r
λ λ λ
t t  2! 
2
t r  t! 
=1 + +  2  + ..... +  r 
λ 2!  λ  r!  λ 
r
∞  t 
M X(t) = ∑  
r =0  λ 

t1 1
∴= µ1' Coefficient
=

=
Mean
ww
µ '2 Coefficient
=
t 2
2
of
1! λ

w.E of
2! λ 2
2
Variance= µ 2 =µ '2 − µ1' 2 = 2 − 2 =
1 1

Variance = 2
1
Mean =
λ λ
1 asy λ2

λ
Example: 1.10.1
λ
En
 1 −3x
 e
gin
Let ‘X’ be a random variable with p.d.f

x>0

Find
F(x)

1) P(X > 3)
= 3
0
 ee
otherwise
2) MGF of ‘X’ rin
Solution
WKT the exponential distribution is g.n
F(x) = λe-λx, x > 0
Here λ =
1
3
et
∞ ∞ 1 −x
P(x>3) = ∫ f (x) dx = ∫ e 3 dx
3 3 3
P(X>3) = e-1
λ
MGF is M X (t) =
λ−t

30

Downloaded From : www.EasyEngineering.net


Downloaded From : www.EasyEngineering.net

MA6451 PROBABILITY AND RANDOM PROCESSES

1 1
3 3 1
= = =
1 1 − 3t 1 − 3t
−t
3 3
1
M X(t) =
1 − 3t
Note
If X is exponentially distributed, then
P(X > s+t / x > s) = P(X > t), for any s, t > 0.

1.11 GAMMA DISTRIBUTION


Definition

ww A Continuous random variable X taking non-negative values is said to follow gamma


distribution , if its probability density function is given by

w.E f(x) = , α>0, 0 < x < ∞


=0, elsewhere

and = asy dx

=0, elsewhere En
gin
When α is the parameter of the distribution.

If X 1 ,X 2 , X 3,.... X k
ee
Additive property of Gamma Variates
are
rin
independent gamma variates with parameters
λ 1, λ 2,….. λ k respectively then X1+X2 + X 3+.... +X k is also a gamma variates with parameter λ 1 + λ 2
+ ….. + λ k
. g.n
Example :1.11.1
et
Customer demand for milk in a certain locality ,per month , is Known to be a
general Gamma RV.If the average demand is a liters and the most likely demand b liters (b<a)
, what is the varience of the demand?
Solution :
Let X be represent the monthly Customer demand for milk.
Average demand is the value of E(X).
Most likely demand is the value of the mode of X or the value of X for which its density
function is maximum.
If f(x) is the its density function of X ,then

f(x) = . xk-1 e-λx e-λx , x>0

31

Downloaded From : www.EasyEngineering.net


Downloaded From : www.EasyEngineering.net

MA6451 PROBABILITY AND RANDOM PROCESSES

f(x) = .[(k-1) xk-2 e-λx - e-λx ]

= 0 ,when x=0 , x=

f” (x) = .[(k-1) xk-2 e-λx - e-λx ]


<0 , when x=
Therefour f(x) is maximum , when x=
i.e ,Most likely demand = =b ….(1)

ww and E(X) = ………(2)

w.E Now V(X) = =


= a (a-b)
=
From (1) and (2)

asy
En
TUTORIAL QUESTIONS

gin
1.It is known that the probability of an item produced by a certain
machine will be defective is 0.05. If the produced items are sent to the
market in packets of 20, fine the no. of packets containing at least,

ee
exactly and atmost 2 defective items in a consignment of 1000 packets
using (i) Binomial distribution (ii) Poisson approximation to binomial
distribution.
rin
2. The daily consumption of milk in excess of 20,000 gallons is
approximately exponentially distributed with . 3000 = θ The city has a
daily stock of 35,000 gallons. What is the probability that of two days g.n
selected at random, the stock is insufficient for both days.
et
3.The density function of a random variable X is given by f(x)= KX(2-X), 0≤X≤2.Find K, mean,
variance and rth moment.
4.A binomial variable X satisfies the relation 9P(X=4)=P(X=2) when n=6. Find the parameter p
of the Binomial distribution.
5. Find the M.G.F for Poisson Distribution.
6. If X and Y are independent Poisson variates such that P(X=1)=P(X=2) and P(Y=2)=P(Y=3).
Find V(X-2Y).
7.A discrete random variable has the following probability distribution
X: 0 1 2 3 4 5 6 7 8
P(X) a 3a 5a 7a 9a 11a 13a 15a 17a
Find the value of a, P(X<3) and c.d.f of X.

32

Downloaded From : www.EasyEngineering.net


Downloaded From : www.EasyEngineering.net

MA6451 PROBABILITY AND RANDOM PROCESSES

7. In a component manufacturing industry, there is a small probability of 1/500 for any


component to be defective. The components are supplied in packets of 10. Use Poisson
distribution to calculate the approximate number of packets containing (1). No defective. (2).
Two defective components in a consignment of 10,000 packets.

WORKED OUT EXAMPLES

Example :1
Given the p.d.f. of a continuous random variable ‘X’ follows
6x(1 − x), 0 < x <1
f (x) =  , find c.d.f. for ‘X’

ww
Solution
0

6x(1 − x),
otherwise

0 < x <1
w.E Given f (x) = 
0
x
otherwise

=
The c.d.f is F(x)
(i) When x < 0, then
−∞
asy
∫ f (x) dx , − ∞ < x < ∞

F(x)
x
= ∫ f (x) dx En
= ∫ 0 dx
−∞
x
=0 gin
F(x)
−∞
(ii) When 0< x < 1, then
x
= ∫ f (x) dx
ee rin
g.n
−∞
0 x
= ∫ f (x) dx + ∫ f (x) dx
−∞

0
x
0

0
x  x 2 x3 
= 0 + ∫ 6x(1 − x) dx = 6 ∫ x(1 − x) dx = 6  − 
2 3 0
x
et
= 3x 2 − 2x 3
(iii) When x > 1, then
x
F(x) = ∫ f (x) dx
−∞
0 1 x
= ∫ 0dx + ∫ 6x(1 − x) dx + ∫ 0 dx
−∞ 0 0
1
6 ∫ (x − x 2 ) dx =1
0

33

Downloaded From : www.EasyEngineering.net


Downloaded From : www.EasyEngineering.net

MA6451 PROBABILITY AND RANDOM PROCESSES

Using (1), (2) & (3) we get


0, x<0

= 3x 2 − 2x 3 ,
F(x) 0 < x <1
1, x >1

Example :2
A random variable X has the following probability function
Values of X 0 1 2 3 4 5 6 7 8
Probability P(X) a 3a 5a 7a 9a 11a 13a 15a 17a

ww (i)
(ii)
(iii)
Determine the value of ‘a’
Find P(X<3), P(X≥3), P(0<X<5)
Find the distribution function of X.

w.E
Solution
Table 1
Values of X 0 1 2 3 4 5 6 7 8
p(x)
asy
a 3a 5a 7a 9a 11a 13a 15a 17a

∑ p(x ) = 1 En
(i) We know that if p(x) is the probability of mass function then

i =0
i

gin
p(0) + p(1) + p(2) + p(3) + p(4) + p(5) + p(6) + p(7) + p(8) = 1

81 a =
a =
1
1/81
ee
a + 3a + 5a + 7a + 9a + 11a + 13a + 15a + 17a = 1

rin
put a = 1/81 in table 1, e get table 2
Table 2
g.n
X=x 0
P(x)
1 2 3 4 5 6
1/81 3/81 5/81 7/81 9/81 11/81 13/81 15/81 17/81
7
et 8

(ii) P(X < 3) = p(0) + p(1) + p(2)


= 1/81+ 3/81 + 5/81 = 9/81
(ii) P(X ≥ 3) = 1 - p(X < 3)
= 1 - 9/81 = 72/81
(iii) P(0 < x < 5) = p(1) + p(2) + p(3) + p(4) here 0 & 5 are not include
= 3/81 + 5/81 + 7/81 + 9/81
3+5+7+8+9 24
= ––––––––––––––– = –––––
81 81
(iv) To find the distribution function of X using table 2, we get
34

Downloaded From : www.EasyEngineering.net


Downloaded From : www.EasyEngineering.net

MA6451 PROBABILITY AND RANDOM PROCESSES

X=x F(X) = P(x ≤ x)


0 F(0) = p(0) = 1/81
F(1) = P(X ≤ 1) = p(0) + p(1)
1
= 1/81 + 3/81 = 4/81
F(2) = P(X ≤ 2) = p(0) + p(1) + p(2)
2
= 4/81 + 5/81 = 9/81
F(3) = P(X ≤ 3) = p(0) + p(1) + p(2) + p(3)
3
= 9/81 + 7/81 = 16/81
F(4) = P(X ≤ 4) = p(0) + p(1) + …. + p(4)
4

5 ww = 16/81 + 9/81 = 25/81


F(5) = P(X ≤ 5) = p(0) + p(1) + ….. + p(4) + p(5)

6 w.E = 2/81 + 11/81 = 36/81


F(6) = P(X ≤ 6) = p(0) + p(1) + ….. + p(6)
= 36/81 + 13/81 = 49/81

7
F(7) asy
= P(X ≤ 7) = p(0) + p(1) + …. + p(6) + p(7)

F(8) En
= 49/81 + 15/81 = 64/81
= P(X ≤ 8) = p(0) + p(1) + ….. + p(6) + p(7) + p(8)
8
gin
= 64/81 + 17/81 = 81/81 = 1

Example :3
ee rin
The mean and SD of a binomial distribution are 5 and 2, determine the distribution.
Solution
Given
SD = npq = 2
Mean = np = 5
(2)
(1)

g.n
(2)
(1)

4 1
∴ p =1 − =
np
npq 5

4
=⇒ q=

p=
1
4
5 et
5 5 5
Sub (3) in (1) we get
n x 1/5 = 5
n = 25
∴ The binomial distribution is
P(X = x) = p(x) = nC x px qn-x
= 25C x(1/5) (4/5)n-x,
x
x = 0, 1, 2, ….., 25
Example :4
If X is a Poisson variable

35

Downloaded From : www.EasyEngineering.net


Downloaded From : www.EasyEngineering.net

MA6451 PROBABILITY AND RANDOM PROCESSES

P(X = 2) = 9 P(X = 4) + 90 P(X=6)


Find (i) Mean if X (ii) Variance of X
Solution
e −λ λ x
P(X=x) = , x = 0,1, 2,.....
x!
Given P(X = 2) = 9 P(X = 4) + 90 P(X=6)
e −λ λ 2 e −λ λ 4 e −λ λ 6
=9 + 90
2! 4! 6!
1 9λ 90λ 2 4
= +
2 4! 6!
1 3λ 2
λ 4
+
ww 2
=


8
2
+
λ
8
4

w.E
1=
4 4
λ 4 + 3λ 2 − 4 =0
λ2 = 1
λ =±1
or
or
λ2 = -4
λ = ± 2i asy


Mean = λ = 1, Variance = λ = 1
Standard Deviation = 1
En
gin
ee rin
g.n
et

36

Downloaded From : www.EasyEngineering.net


Downloaded From : www.EasyEngineering.net

MA6451 PROBABILITY AND RANDOM PROCESSES

UNIT – II

TWO DIMENSIONAL RANDOM VARIABLES

Introduction

In the previous chapter we studied various aspects of the theory of a single R.V. In this
chapter we extend our theory to include two R.V's one for each coordinator axis X and Y
of the XY Plane.

DEFINITION : Let S be the sample space. Let X = X(S) & Y = Y(S) be two functions each
assigning a real number to each outcome s ∈ S. hen (X, Y) is a two dimensional random
variable.

ww
2.1 Types of random variables
1. Discrete R.V.’s

w.E
2. Continuous R.V.’s
Discrete R.V.’s (Two Dimensional Discrete R.V.’s)
If the possible values of (X, Y) are finite, then (X, Y) is called a two dimensional discrete

asy
R.V. and it can be represented by (x i , y), i = 1,2,….,m.
In the study of two dimensional discrete R.V.’s we have the following

En
5 important terms.
• Joint Probability Function (JPF) (or) Joint Probability Mass Function.
• Joint Probability Distribution.

gin
• Marginal Probability Function of X.
• Marginal Probability Function of Y.
• Conditional Probability Function.
ee
2.1.1 Joint Probability Function of discrete R.V.’s X and Y
rin
g.n
The function P(X = x i , Y = y j ) = P(x i , y j ) is called the joint probability function for
discrete random variable X and Y is denote by p ij .
Note
1. P(X = xi , Y = yj ) = P[(X = xi )∩(Y = yj )] = p ij
2. It should satisfies the following conditions
(i) p ij ≥ ∀ i, j (ii) Σ j Σ i p ij = 1
et
2.1.2 Marginal Probability Function of X
If the joint probability distribution of two random variables X and Y is given then the
marginal probability function of X is given by
P x(x i ) = p i (marginal probability function of Y)
Conditional Probabilities
The conditional probabilities function of X given Y = yj is given by
P[X = xi / Y = yj ] p ij
P[X = xi / Y = yj ] = –––––––––––––––– = ––––
P[Y = yj ] p .j

37

Downloaded From : www.EasyEngineering.net


Downloaded From : www.EasyEngineering.net

MA6451 PROBABILITY AND RANDOM PROCESSES

The set {x i , p ij / p .j }, i = 1, 2, 3, …..is called the conditional probability distribution of X


given Y = yj .
The conditional probability function of Y given X = xi is given by
P[Y = yi / X = xj ] p ij
P[Y = yi / X = xj ] = –––––––––––––––– = ––––
P[X = xj ] pi.
The set {yi , p ij / p i. }, j = 1, 2, 3, …..is called the conditional probability distribution of Y
given X = x i .

SOLVED PROBLEMS ON MARGINAL DISTRIBUTION


Example:2.1.1
From the following joint distribution of X and Y find the marginal distributions.
X

ww Y
0
0
3/28
1
9/28
2
3/28

w.E 1
2
3/14
1/28
3/14
0
0
0
Solution
X
Y asy 0 2 P Y (y) = p(Y=y)
0
En3/28 P(0,0) 3/28 P(2,0) 15/28 = P y (0)
1
2 gin
3/14 P(0, 1)
1/28 P(0,2)
3/14 P(1,1)
0 P(2,2)
6/14 = P y (1)
1/28 = P y (2)
P X (X) = P(X=x)

The marginal distribution of X


ee
10/28 = 5/14
P X (0)
3/28
P X (2)
1

rin
P X (0) = P(X = 0) = p(0,0) + p(0,1) + p(0,2) = 5/14
P X (1) = P(X = 1) = p(1,0) + p(1,1) + p(1,2) = 15/28 g.n
P X (2) = P(X = 2) = p(2,0) + p(2,1) + p(2,2) = 3/28
Marginal probability function of X is
5
et
14 , x = 0

 15
=
PX (x) = , x 1
 28
3
 28 , x = 2

The marginal distribution of Y
P Y (0) = P(Y = 0) = p(0,0) + p(1,0) + p(2,0) = 15/28
P Y (1) = P(Y = 1) = p(0,1) + p(2,1) + p(1,1) = 3/7

38

Downloaded From : www.EasyEngineering.net


Downloaded From : www.EasyEngineering.net

MA6451 PROBABILITY AND RANDOM PROCESSES

P Y (2) = P(Y = 2) = p(0,2) + p(1,2) + p(2,2) = 1/28


Marginal probability function of Y is

 15
 28 , y = 0

3
=
PY (y) = , y 1
7
1
 28 , y = 2

2.3 CONTINUOUS RANDOM VARIABLES

ww • Two dimensional continuous R.V.’s


If (X, Y) can take all the values in a region R in the XY plans then (X, Y) is called two-
dimensional continuous random variable.

w.E
• Joint probability density function :
∞ ∞
(i) f XY (x,y) ≥0 ; (ii) ∫ ∫ f XY (x, y) dydx =1

• asy
−∞ −∞
Joint probability distribution function
F(x,y) = P[X ≤ x, Y ≤ y]
x
y
= ∫  ∫ f (x, y)dx  dy En 


−∞  −∞

gin

Marginal probability density function

−∞

ee
f(x) = fX (x) = ∫ f x,y (x, y)dy (Marginal pdf of X)

f(y) = fY (x) = ∫ f x,y (x, y)dy (Marginal pdf of Y) rin



−∞
Conditional probability density function g.n
(i) P(Y= y / X= x)
= f (y / x) =
f (x, y)
f (x)
f (x, y)
, f (x) > 0
et
(ii) P(X= x / Y= y)
= f (x / y) = , f (y) > 0
f (y)

Example :2.3.1
2
 (2x + 3y), 0 < x < 1, 0 < y <1
Show that the function f (x, y) =  5
0 otherwise
is a joint density function of X and Y.
Solution
39

Downloaded From : www.EasyEngineering.net


Downloaded From : www.EasyEngineering.net

MA6451 PROBABILITY AND RANDOM PROCESSES

We know that if f(x,y) satisfies the conditions


∞ ∞
(i) f (x, y) ≥ 0 (ii) ∫ ∫ f (x, y) = 1 , then f(x,y) is a jdf
−∞ −∞

2
 (2x + 3y), 0 < x < 1, 0 < y <1
Given f (x, y) =  5
0 otherwise
(i) f (x, y) ≥ 0 in the given interval 0 ≤ (x,y) ≤ 1
∞ ∞ 11 2
(ii) ∫ ∫ f (x, y) dx dy = ∫ ∫ (2x + 3y) dx dy
−∞ −∞ 00 5
1
2 1 1  x2 
= ∫ ∫ 2 + 3xy  dy

ww 5 00 2
21
0
2
= ∫ (1 + 3y) dy =  y +
3y 2  2 3
1

= 1 + 
w.E 50
2 5
=   =1
5 
2 0 5  2 

asy
5 2
Since f(x,y) satisfies the two conditions it is a j.d.f.
Example :2.3.2
En
The j.d.f of the random variables X and Y is given
8xy,
f (x, y) = 
0,
0 < x < 1,
otherwise gin
0<y<x

Find (i) f X(x)


Solution
We know that
(ii) fY (y)
ee
(iii) f(y/x)

rin
(i) The marginal pdf of ‘X’ is

=
f X (x) = f(x) = ∫ f (x, y)dy
x
∫=
8xy dy 4x 3 g.n
= 4x , 0 < x < 1
f (x) 3
−∞

(ii) The marginal pdf of ‘Y’ is


0

et
∞ 1
=
fY (y) = f(y) = ∫ f (x, y)dy ∫=
8xy dy 4y
−∞ 0
f=
(y) 4y, 0 < y < α
(iii) We know that
f (x, y)
f (y / x) =
f (x)
8xy 2y
= 3
= , 0 < y < x, 0 < x < 1
4x x2
40

Downloaded From : www.EasyEngineering.net


Downloaded From : www.EasyEngineering.net

MA6451 PROBABILITY AND RANDOM PROCESSES

Result
Marginal pdf g Marginal pdf y F(y/x)
2y
4x3, 0<x<1 4y, 0<y<x ,0 < y < x, 0 < x < 1
x2

2.4 REGRESSION
* Line of regression
The line of regression of X on Y is given by
σy
x=
− x r. (y − y)
σx
The line of regression of Y on X is given by
σy
ww y=
− y r.
σx
(x − x)
* Angle between two lines of Regression.

w.E tan θ =
1 − r 2  σyσx

r  σ x 2 + σ y2



* Regression coefficient

asy 

r.
σy
= b YX En
Regression coefficients of Y on X

σx
gin
Regression coefficient of X and Y

r.
σx
σy
= b XY

∴ Correlation coefficient r =
± b XY × b YX
ee rin
g.n
Example:2.4.1
1. From the following data, find
(i) The two regression equation
(ii) The coefficient of correlation between the marks in Economic and Statistics.
et
(iii) The most likely marks in statistics when marks in Economic are 30.

Marks in Economics 25 28 35 32 31 36 29 38 34 32
Marks in Statistics 40 46 49 41 36 32 31 30 33 39

Solution

41

Downloaded From : www.EasyEngineering.net


Downloaded From : www.EasyEngineering.net

MA6451 PROBABILITY AND RANDOM PROCESSES

(X − X) (Y − Y) (X − X) (Y − Y)
2 2 2
X Y X − X = X − 32 X − Y = Y − 38
25 43 -7 5 49 25 -35
28 46 -4 8 16 64 -32
35 4 3 11 9 121 33
32 41 0 3 0 9 0
31 36 -1 -2 1 4 2
36 32 4 -6 16 36 -24
29 31 -3 -7 09 49 +21
38 30 6 -8 36 64 -48
34 33 2 -5 4 25 -48
32 39 0 1 0 1 100

ww
320 380 0 0 140 398 -93

∑ X 320 ∑ Y 380
Here = = = 32 and=
Y = = 38
w.E
X
n 10 n 10
Coefficient of regression of Y on X is
∑ (X − X)(Y − Y) −93
b YX =
∑ (X − X) asy
2
=
140
= − 0.6643

b XY = =En
Coefficient of regression of X on Y is
∑ (X − X)(Y − Y) −93
= − 0.2337
∑ (Y − Y)
2

gin
398
Equation of the line of regression of X and Y is
X−X
X – 32
X
=b XY (Y − Y)
ee
= -0.2337 (y – 38)
= -0.2337 y + 0.2337 x 38 + 32 rin
X = -0.2337 y + 40.8806
Equation of the line of regression of Y on X is g.n
Y−Y
Y – 38
Y
=b YX (X − X)
= -0.6643 (x – 32)
= -0.6643 x + 38 + 0.6643 x 32
et
= -0.6642 x + 59.2576
Coefficient of Correlation
r2 = bYX × b XY
= -0.6643 x (-0.2337)
r = 0.1552
r = ± 0.1552
r = ± 0.394
Now we have to find the most likely mark, in statistics (Y) when marks in economics (X) are 30.
y = -0.6643 x + 59.2575
42

Downloaded From : www.EasyEngineering.net


Downloaded From : www.EasyEngineering.net

MA6451 PROBABILITY AND RANDOM PROCESSES

Put x = 30, we get


y = -0.6643 x 30 + 59.2536
= 39.3286
y~ 39

2.5 COVARIANCE
Def : If X and Y are random variables, then Covariance between X and Y is defined as
Cov (X, Y) = E(XY) – E(X) . E(Y)
Cov (X, Y) = 0 [If X & Y are independent]

2.6 CORRELATION
Types of Correlation
• Positive Correlation

• ww (If two variables deviate in same direction)


Negative Correlation

w.E (If two variables constantly deviate in opposite direction)

2.7 KARL-PEARSON’S COEFFICIENT OF CORRELATION

asy
Correlation coefficient between two random variables X and Y usually denoted by r(X,
Y) is a numerical measure of linear relationship between them and is defined as
Cov(X, Y)
r(X, Y) =
σ X .σ Y
En
,

1
Where Cov (X, Y) = ∑ XY − X Y
n gin
= σX
∑X
n
; = σY
∑Y

* Limits of correlation coefficient


n ee rin
-1 ≤ r ≤ 1.
X & Y independent, ∴ r(X, Y) = 0.
g.n
Note :Types of correlation based on ‘r’.
Values of ‘r’
r=1
0<r<1
Correlation is said to be
perfect and positive
positive
et
-1<r<0 negative
r=0 Uncorrelated

SOLVED PROBLEMS ON CORRELATION


Example :2.6.1
Calculated the correlation coefficient for the following heights of fathers X and their sons
Y.
X 65 66 67 67 68 69 70 72
Y 67 68 65 68 72 72 69 71
43

Downloaded From : www.EasyEngineering.net


Downloaded From : www.EasyEngineering.net

MA6451 PROBABILITY AND RANDOM PROCESSES

Solution
X Y U = X – 68 V = Y – 68 UV U2 V2
65 67 -3 -1 3 9 1
66 68 -2 0 0 4 0
67 65 -1 -3 3 1 9
67 68 -1 0 0 1 0
68 72 0 4 0 0 16
69 72 1 4 4 1 16

ww
70
72
69
71
∑U = 0
2
4
∑V = 0
1
3
2
12
∑ UV = 24 ∑ U 2 = 36 ∑ V 2 = 52
4
16
1
9

Now
w.E ∑U
U= =
0
= 0

V=
n
∑V
=
8
8
= 1
asy
n 8
Cov (X, Y) = Cov (U, V) En

∑ UV
− UV =
24
−0= 3 gin (1)

σ=
U
n
∑U
n
2
2
− U=
8
36
8
ee
− 0= 2.121
rin
(2)

σ=
V
∑V
n
2
2
− V=
52
8
− 1= 2.345 g.n
(3)

∴r(X, Y) = r(U, V) =

= 0.6031
Cov(U, V)
σ U .σ V
=
3
2.121 x 2.345
(by 1, 2, 3)
et
Example :2.6.2
1
Let X be a random variable with p.d.f. f (x) = , − 1 ≤ x ≤1 and let
2
Y = x2, find the correlation coefficient between X and Y.

Solution

44

Downloaded From : www.EasyEngineering.net


Downloaded From : www.EasyEngineering.net

MA6451 PROBABILITY AND RANDOM PROCESSES


E(X) = ∫ x.f (x) dx
−∞
1
1 1 1  x2  11 1
= ∫ x. dx =   =  − = 0
−1 2 2  2  −1 2 2 2
E(X) = 0

E(Y) = ∫ x 2 .f (x) dx
−∞
1
1 1 1  x3  11 1 1 2 1
= ∫ x . dx =  
2
=  + = . =
−1 2 2  3  −1 23 3 2 3 3
E(XY) = E(XX2)

ww = E(X )= ∫ x .f (x)
−∞
3
=
 x4 

dx = 3

 4  −1
0
1

w.E E(XY) = 0
∴r(X, Y) = ρ(X, Y) =
Cov(X, Y)
=0

ρ = 0. asy
σX σY

En
Note : E(X) and E(XY) are equal to zero, noted not find σ x &σ y .

gin
2.8 TRANSFORMS OF TWO DIMENSIONAL RANDOM VARIABLE
Formula:

f U (u) = ∫ f u,v (u, v) dv
−∞

ee rin
& f V (u) = ∫ f u,v (u, v) du
−∞

∂ (x, y)
g.n
Example : 1
f UV (u, V) = f XY (x, y)
∂ (u, v) et
If the joint pdf of (X, Y) is given by fxy (x, y) = x+y, 0 ≤ x, y ≤ 1, find the pdf of ∪ = XY.

Solution
Given f xy (x, y) = x + y
Given U = XY
Let V=Y
u
=x =&y V
v

45

Downloaded From : www.EasyEngineering.net


Downloaded From : www.EasyEngineering.net

MA6451 PROBABILITY AND RANDOM PROCESSES

∂x 1 ∂x − u ∂y ∂y
= = . =; 0;= 1 (1)
∂u V ∂v V ∂u2
∂v
∂y ∂x
1 −u
∂ (x, y) ∂u ∂v 1 1
=
∴J = = V V2 = −1 =
∂ (u, v) ∂y ∂y v v
0 1
∂u ∂v
1
⇒|J|= (2)
V
The joint p.d.f. (u, v) is given by
f uv (u, v) = f xy (x, y) |J|

ww 1 u
1
= (x + y)
|v|

w.E
The range of V :
=  + u
Vv 
(3)

The range of u : asy


Since 0 ≤ y ≤ 1, we have 0 ≤ V ≤ 1 (∴ V = y)

Given

0≤x≤1
0 ≤
u
≤ En
⇒ 0≤u≤v
v
1
gin
Hence the p.d.f. of (u, v) is given by

f uv (u, v) =
1u  ee
 + v  , 0 ≤ u ≤ v, 0 ≤ v ≤ 1
v v  rin
Now

f U (u) = ∫ f u,v (u, v) dv g.n
−∞
1
= ∫ f u,v (u, v) dv
u
et
 u 1

= ∫  2 + 1 dv
u v 
1
 v −1 
=  v + u. 
 −1  u
∴fu (u) = 2(1-u), 0 < u < 1
p.d.f of (u, v) p.d.f of u = XY

46

Downloaded From : www.EasyEngineering.net


Downloaded From : www.EasyEngineering.net

MA6451 PROBABILITY AND RANDOM PROCESSES

1u 
=
f uv (u, v)  + v fu (u) = 2(1-u), 0 < u < 1
v v 
0 ≤ u ≤ v, 0 ≤ v ≤ 1

TUTORIAL QUESTIONS

1. The jpdf of r.v X and Y is given by f(x,y)=3(x+y),0<x<1,0<y<1,x+y<1 and 0 otherwise. Find


the marginal pdf of X and Y and ii) Cov(X,Y).
2. Obtain the correlation coefficient for the following data:
X: 68 64 75 50 64 80 75 40 55 64
Y: 62 58 68 45 81 60 48 48 50 70

ww
3.The two lines of regression are 8X-10Y+66=0, 4X-18Y-214=0.The variance of x is 9 find i)
The mean value of x and y. ii) Correlation coefficient between x and y.
4. If X 1 ,X 2 ,…X n are Poisson variates with parameter λ=2, use the central limit theorem to find

w.E
P(120≤S n ≤160) where Sn=X 1 +X 2 +…X n and n=75.
5. If the joint probability density function of a two dimensional random variable (X,Y) is
given by f(x, y) = x2 + , 0<x<1,0<y<2= 0, elsewhere Find (i) P(X>1/2)(ii) P(Y<X) and (iii)
P(Y<1/2/ X<1/2).
asy
6. Two random variables X and Y have joint density

En Find Cov (X,Y).

gin
7. If the equations of the two lines of regression of y on x and x on y are respectively
7x-16y+9=0; 5y-4x-3=0, calculate the coefficient of correlation.

WORKEDOUT EXAMPLES
Example 1 ee
The j.d.f of the random variables X and Y is given rin
8xy,
f (x, y) = 
0,
0 < x < 1,
otherwise
0<y<x
g.n
Find (i) f X(x)
Solution
We know that
(ii) fY (y) (iii) f(y/x)
et
(i) The marginal pdf of ‘X’ is
∞ x
=
f X (x) = f(x) = ∫ f (x, y)dy ∫=
8xy dy 4x 3
−∞ 0

= 4x , 0 < x < 1
f (x) 3

(ii) The marginal pdf of ‘Y’ is


∞ 1
=
fY (y) = f(y) = ∫ f (x, y)dy ∫=
8xy dy 4y
−∞ 0
f=
(y) 4y, 0 < y < α
47

Downloaded From : www.EasyEngineering.net


Downloaded From : www.EasyEngineering.net

MA6451 PROBABILITY AND RANDOM PROCESSES

(iii) We know that


f (x, y)
f (y / x) =
f (x)
8xy 2y
= 3
= , 0 < y < x, 0 < x < 1
4x x2
Example 2
1
Let X be a random variable with p.d.f. f (x) = , − 1 ≤ x ≤1 and let
2
Y = x2, find the correlation coefficient between X and Y.

Solution

ww E(X) = ∫ x.f (x) dx


−∞

1  x2 
1

w.E −1
11
= ∫ x. dx
2
=  
2  2  −1
=
11 1
 − = 0
2 2 2

asy
E(X) = 0

E(Y) = ∫ x 2 .f (x) dx
−∞

1 1 1  x3 
= ∫ x . dx =  
2 En 1

=
11 1 1 2 1
 + = . =
−1

E(XY) = E(XX2)
2 2  3  −1
gin 23 3 2 3 3

= E(X )= ∫ x .f (x)
−∞
3
=
 x4 
dx =


 4  −1
3
0 ee 1

rin
E(XY) = 0
∴r(X, Y) = ρ(X, Y) =
Cov(X, Y)
=0 g.n
ρ = 0.
σX σY

Note : E(X) and E(XY) are equal to zero, noted not find σ x &σ y .
et
Result
Marginal pdf g Marginal pdf y F(y/x)
2y
4x3, 0<x<1 4y, 0<y<x ,0 < y < x, 0 < x < 1
x2

48

Downloaded From : www.EasyEngineering.net


Downloaded From : www.EasyEngineering.net

MA6451 PROBABILITY AND RANDOM PROCESSES

UNIT - III
RANDOM PROCESSES

Introduction
In chapter 1, we discussed about random variables. Random variable is a function of the
possible outcomes of a experiment. But, it does not include the concept of time. In the
real situations, we come across so many time varying functions which are random in
nature. In electrical and electronics engineering, we studied about signals.
Generally, signals are classified into two types.
(i) Deterministic
(ii) Random
Here both deterministic and random signals are functions of time. Hence it is

ww
possible for us to determine the value of a signal at any given time. But this is not
possible in the case of a random signal, since uncertainty of some element is always
associated with it. The probability model used for characterizing a random signal is called
w.E
a random process or stochastic process.

3.1 RANDOM PROCESS CONCEPT


asy
A random process is a collection (ensemble) of real variable {X(s, t)} that are functions
of a real variable t where s ∈ S, S is the sample space and
t ∈T. (T is an index set).
En
REMARK
gin
i) If t is fixed, then {X(s, t)} is a random variable.

ee
ii) If S and t are fixed {X(s, t)} is a number.
iii) If S is fixed, {X(s, t)} is a signal time function.

rin
NOTATION

g.n
Here after we denote the random process {X(s, t)} by {X(t)} where the index set T is assumed to
be continuous process is denoted by {X(n)} or {Xn}.

A comparison between random variable and random process


et
Random Variable Random Process
A function of the possible outcomes of A function of the possible outcomes of
an experiment is X(s) an experiment and also time i.e, X(s, t)
Outcomes are mapped into wave from
Outcome is mapped into a number x.
which is a fun of time 't'.

49

Downloaded From : www.EasyEngineering.net


Downloaded From : www.EasyEngineering.net

MA6451 PROBABILITY AND RANDOM PROCESSES

3.2 CLASSIFICATION OF RANDOM PROCESSES


We can classify the random process according to the characteristics of time t and the
random variable X = X(t) t & x have values in the ranges
–∞< t <∞ and –∞< x <∞.

Random Process is a function of

ww
Random Variables Time t

w.E
Discrete
asy
Continuous Discrete Continuous

En
3.2.1 CONTINUOUS RANDOM PROCESS
gin
If 'S' is continuous and t takes any value, then X(t) is a continuous random variable.
Example
ee
Let X(t) = Maximum temperature of a particular place in (0, t). Here 'S' is a continuous
set and t ≥ 0 (takes all values), {X(t)} is a continuous random process.
rin
3.2.2 DISCRETE RANDOM PROCESS
g.n
If 'S' assumes only discrete values and t is continuous then we call such random process
{X(t) as Discrete Random Process.
Example
Let X(t) be the number of telephone calls received in the interval (0, t).
Here, S = {1, 2, 3, …}
et
T = {t, t ≥ 0}
∴ {X(t)} is a discrete random process.

3.2.3 CONTINUOUS RANDOM SEQUENCE


If 'S' is a continuous but time 't' takes only discrete is called discrete random sequence.
Example: Let X n denote the outcome of the nth toss of a fair die.
Here S = {1, 2, 3, 4, 5, 6}
T = {1, 2, 3, …}
∴ (X n , n = 1, 2, 3, …} is a discrete random sequence.
50

Downloaded From : www.EasyEngineering.net


Downloaded From : www.EasyEngineering.net

MA6451 PROBABILITY AND RANDOM PROCESSES

3.3 CLASSIFICATION OF RANDOM PROCESSES BASED ON ITS SAMPLE


FUNCTIONS
Non-Deterministic Process
A Process is called non-deterministic process if the future values of any sample function
cannot be predicted exactly from observed values.

Deterministic Process
A process is called deterministic if future value of any sample function can be predicted
from past values.

3.3.1 STATIONARY PROCESS


A random process is said to be stationary if its mean, variance, moments etc are constant.

ww
Other processes are called non stationary.

1. 1st Order Distribution Function of {X(t)}

w.E For a specific t, X(t) is a random variable as it was observed earlier.


F(x, t) = P{X(t) ≤ x} is called the first order distribution of the process {X(t)}.

f ( x, t ) =
∂ asy
1st Order Density Function of {X(t)}
F ( x, t ) is called the first order density of {X, t}
∂x
En
2nd Order distribution function of {X(t)}
gin
F ( x1 , x 2 ; t1 , t 2 ) =P {X ( t1 ) ≤ x1;X ( t 2 ) ≤ x 2 } is the point distribution of the random

2nd order density function of {X(T)}


ee
variables X(t 1 ) and X(t 2 ) and is called the second order distribution of the process {X(t)}.

∂ 2 F ( x1 , x 2 ; t 1 , t 2 ) rin
f ( x1 , x 2 ; t 1 , t 2 ) =
∂x, ∂x 2
g.n
is called the second order density of {X(t)}.

3.3.2 First - Order Stationary Process


Definition et
A random process is called stationary to order, one or first order stationary if its 1st order
density function does not change with a shift in time origin.
In other words,
( x1 , t1 ) f X ( x1 , t1 + C ) must be true for any t 1 and any real number C if {X(t 1 )} is to
f X=
be a first order stationary process.

Example :3.3.1
Show that a first order stationary process has a constant mean.
Solution
Let us consider a random process {X(t 1 )} at two different times t 1 and t 2 .
51

Downloaded From : www.EasyEngineering.net


Downloaded From : www.EasyEngineering.net

MA6451 PROBABILITY AND RANDOM PROCESSES


∴ E  X ( t1 )  = xf ( x, t1 )dx

−∞
[f(x,t 1 ) is the density form of the random process X(t 1 )]

∴ E  X ( t 2 )  = xf ( x, t 2 )dx

−∞
[f(x,t 2 ) is the density form of the random process X(t 2 )]
Let t 2 = t 1 + C
∞ ∞
 X ( t 2 ) 
∴E= ∫ xf ( x, t=
+ C )dx ∫ xf ( x, t )dx
1 1
−∞ −∞

= E  X ( t1 ) 

ww
Thus E  X ( t 2 )  =E  X ( t1 ) 
Mean process {X(t1)} = mean of the random process {X(t 2 )}.

w.E
Definition 2:
If the process is first order stationary, then
Mean = E(X(t)] = constant
3.3.4 Second Order Stationary Process
asy
A random process is said to be second order stationary, if the second order density
function stationary.
f ( x1 , x 2 ;= En
t1 , t 2 ) f ( x1 , x 2 ; t1 + C, t 2 + C ) ∀x1 , x 2 and C.

gin
E ( X12 ) , E ( X 22 ) , E ( X1 , X 2 ) denote change with time, where
X = X(t 1 ); X2 = X(t 2 ).
3.3.5 Strongly Stationary Process
ee rin
A random process is called a strongly stationary process or Strict Sense Stationary
Process (SSS Process) if all its finite dimensional distribution are invariance under translation of
time 't'.
f X (x 1 , x2 ; t 1 , t 2 ) = fX(x 1 , x2 ; t 1 +C, t 2 +C) g.n
In general

C.
f X (x 1 , x2 , x3 ; t 1 , t 2 , t 3 ) = fX (x 1 , x2 , x 3 ; t 1 +C, t 2 +C, t 3 +C)
et
f X (x 1 , x 2 ..x n ; t 1 , t 2 …t n) = f X(x1 , x2 ..x n ; t 1 +C, t 2 +C..t n +C) for any t 1 and any real number

3.3.6 Jointly - Stationary in the Strict Sense


{X(t)} and Y{(t)} are said to be jointly stationary in the strict sense, if the joint
distribution of X(t) and Y(t) are invariant under translation of time.
Definition Mean:
µ X ( t )= E  X ( t1 )  , −∞ < t < ∞
µ  X ( t )  is also called mean function or ensemble average of the random process.
3.3.7 Auto Correlation of a Random Process

52

Downloaded From : www.EasyEngineering.net


Downloaded From : www.EasyEngineering.net

MA6451 PROBABILITY AND RANDOM PROCESSES

Let X(t 1 ) and X(t 2 ) be the two given numbers of the random process {X(t)}. The auto
correlation is
R XX ( t1 , t 2 ) = E {X ( t1 ) xX ( t 2 )}
Mean Square Value
Putting t 1 = t 2 = t in (1), we get
R XX (t,t) = E[X(t) X(t)]
⇒ R XX ( t, t ) = E  X 2 ( t )  is the mean square value of the random process.
3.3.8 Auto Covariance of A Random Process
{ }
E  X ( t1 ) − E ( X ( t1 ) )   X ( t 2 ) − E ( X ( t 2 ) ) 
C XX ( t1 , t 2 ) =
= R XX ( t1 , t 2 ) − E  X ( t1 )  E  X ( t 2 ) 

ww
Correlation Coefficient
The correlation coefficient of the random process {X(t)} is defined as
C XX ( t1 , t 2 )
w.E ρXX ( t1 , t 2 ) =
Var X ( t1 ) xVar X ( t 2 )
Where C XX (t 1 , t 2 ) denotes the auto covariance.

3.4 CROSS CORRELATION asy


En
The cross correlation of the two random process {X(t)} and {Y(t)} is defined by
R XY (t 1 , t 2 ) = E[X(t 1 ) Y (t 2 )]

3.5 WIDE - SENSE STATIONARY (WSS)


gin
ee
A random process {X(t)} is called a weakly stationary process or covariance stationary
process or wide-sense stationary process if
i) E{X(t)} = Constant
ii) E[X(t) X(t+τ] = R XX (τ) depend only on τ when τ = t 2 - t 1 . rin
REMARKS :
SSS Process of order two is a WSS Process and not conversely. g.n
3.6 EVOLUTIONARY PROCESS
A random process that is not stationary in any sense is called as evolutionary process. et
SOLVED PROBLEMS ON WIDE SENSE STATIONARY PROCESS
Example:3.6.1
Given an example of stationary random process and justify your claim.
Solution:
Let us consider a random process X(t) = A as (wt + θ) where A &ω are custom and 'θ' is
uniformlydistribution random Variable in the interval
(0, 2π).
Since 'θ' is uniformly distributed in (0, 2π), we have

53

Downloaded From : www.EasyEngineering.net


Downloaded From : www.EasyEngineering.net

MA6451 PROBABILITY AND RANDOM PROCESSES

1
 ,0 < C < 2π
f ( θ ) = 2π
0 ,otherwise


∴ E[X(t)] = ∫ X ( t ) f ( θ ) dθ
−∞

1
= ∫ A ω ( ω t + θ ) 2 π dθ
0

A 2π
= sin ( ωt + θ )  0

ww =
A
2π 
Sin ( 2π + ωt ) − Sin ( ωt + 0 ) 

w.E =
A

[Sinωt − sin ωt ]
= 0 constant

asy
Since E[X(t)] = a constant, the process X(t) is a stationary random process.

Example:3.6.2 which are not stationary

En
Examine whether the Poisson process {X(t)} given by the probability law P{X(t)=n] =
e −λt ( λt )
n
, n = 0, 1, 2, ….
gin
Solution
We know that the mean is given by

E  X ( t )  =∑ nPn ( t )
ee rin
n =0

= ∑

ne −λt ( λt )
n
g.n
n =0

= ∑

n
e −λt ( λt )
n et
n =1 n −1
( λt )
∞ n

=e −λt

n =1 n −1
−λt
 λt ( λt ) 2 
=e  + + ...
 0! 1! 

54

Downloaded From : www.EasyEngineering.net


Downloaded From : www.EasyEngineering.net

MA6451 PROBABILITY AND RANDOM PROCESSES

 λt ( λt ) 2 
= ( λt ) e  1 +
−λt
+ + ... 
 1 2 
 
= ( λt ) e e
−λt λt

= λt , depends on t
Hence Poisson process is not a stationary process.
3.7 ERGODIC RANDOM PROCESS
Time Average
The time average of a random process {X(t)} is defined as
T
1
X ( t ) dt
2T −∫T
XT =

ww
Ensemble Average
The ensemble average of a random process {X(t)} is the expected value of the random
variable X at time t

w.E Ensemble Average = E[X(t)]


Ergodic Random Process
{X(t)} is said to be mean Ergodic
If lim X T = µ
T →∞
asy
En
T
1
X ( t ) dt =
T →∞ 2T ∫
⇒ lim µ

gin
−T
Mean Ergodic Theorem
Let {X(t)} be a random process with constant mean µ and let X T be its time average.
Then {X(t)} is mean ergodic if
lim Var X T = 0
T →∞
ee rin
Correlation Ergodic Process
The stationary process {X(t)} is said to be correlation ergodic if the process {Y(t)} is
mean ergodic where g.n
Y(t) = X(t) X(t+λ)
E  y ( t )  = lim YT when YT is the time average of Y(t).
|T| →∞
et
3.8 MARKOV PROCESS
Definition
A random process {X(t)} is said to be markovian if
P  X ( t n +1 ) ≤ X n +1 / X ( n ) + x n , x ( t n −1 ) = x n −1...x ( t 0 = x 0 ) 
P  X ( t n +1 ) ≤ X n +1 / x ( t n ) =
x n 
Where t 0 ≤ t1 ≤ t 2 ≤ ... ≤ t n ≤ t n +1
Examples of Markov Process

55

Downloaded From : www.EasyEngineering.net


Downloaded From : www.EasyEngineering.net

MA6451 PROBABILITY AND RANDOM PROCESSES

1.The probability of raining today depends only on previous weather conditions existed
for the last two days and not on past weather conditions.
2.A different equation is markovian.

Classification of Markov Process

Markov Process

Continuous Discrete Discrete Continuous


Parameter Parameter Parameter Parameter

ww
Markov Process Markov Process Markov Chain Markov Chain

w.E
3.9 MARKOV CHAIN
Definition

asy
We define the Markov Chain as follows
If P{X n = a n/X n-1 = a n-1 , X n-2 = a n-2 , … X 0 = a 0 }

Chains. En
⇒P{X n = a n / X n-1 = a n-1 } for all n. the process {X n }, n = 0, 1, 2… is called as Markov

2.The conditional probability P{X= gin


1.a1, a2, a3, … an are called the states of the Markov Chain.
aj | X n=
−1 = Pij ( n − 1, n ) is called the one step
ai}

ee
n
transition probability from state a i to state a j at the nth step.
3.The tmp of a Markov chain is a stochastic matricx
i) P ij ≥ 0
rin
ii) ΣP ij = 1 [Sum of elements of any row is 1]
g.n
3.10 Poisson Process

et
The Poisson Process is a continuous parameter discrete state process which is very useful
model for many practical situations. It describe number of times occurred. When an experiment
is conducted as a function of time.
Property Law for the Poisson Process
Let λ be the rate of occurrences or number of occurrences per unit time and P n (t) be the
probability of n occurrences of the event in the interval (0, t) is a Poisson distribution with
parameter λt.
e −λt ( λt )
n

i.e. P  X ( t ) = n  = , n = 0,1, 2,...


n!
e −λt ( λt )
n

Pn ( t ) =
n!
56

Downloaded From : www.EasyEngineering.net


Downloaded From : www.EasyEngineering.net

MA6451 PROBABILITY AND RANDOM PROCESSES

Second Order Probability Function of a Homogeneous Poisson Process


P  X ( t 1 )  = n1 =  X ( t 2 ) = n 2  = P  X ( t1 = n1 )  .P  X ( t 2 = n 2 )  /

 X ( t1 ) =n 2  , t 2 > t1
= P  X ( t1 ) =
n1  .P [the even occurs n2 -n times in the interval (t 2 =t 1 )

{λ ( t − t1 )}
−λ ( t 2 − t1 ) n 2 = n1
e −λt1 ( λt1 ) 1 e
n

, n 2 ≥ n1
2
= .
n1 n 2 − n1
 e −λt 2 .λ n 2 .t1n1 ( t 2 − t1 )n 2 − n1 
 , n 2 ≥ n1 
=  n, n 2 − n1 

ww 
0 , otherwise



w.E
3.11SEMI RANDOM TELEGRAPH SIGNAL PROCESS
If N(t) represents the number of occurrence of a specified event in (0, t) and X(t) = (–)N(t),

asy
then {X(t)} is called a semi-random telegraph signal process.
3.11.1 RANDOM TELEGRAPH SIGNAL PROCESS
Definition

i. En
A random telegraph process is a discrete random process X(t) satisfying the following:
X(t) assumes only one of the two possible values 1 or –1 at any time 't'
ii.
iii.
X(0) = 1 or –1 with equal probability 1/2
gin
The number of occurrence N(t) from one value to another occurring in any interval of

P  N ( t ) = r  =
e −λt ( λt )
r!
r
ee
length 't' is a Poisson process with rate λ, so that the probability of exactly 'r' transitions is

, r = 0,1, 2,...
rin
A typical sample function of telegraph process.
(0,1) g.n
et
(0,-1)

Note: The process is an example for a discrete random process.


* Mean and Auto Correlation P{X(t) = 1} and P{X(t) = 1" for any t.

57

Downloaded From : www.EasyEngineering.net


Downloaded From : www.EasyEngineering.net

MA6451 PROBABILITY AND RANDOM PROCESSES

TUTORIAL QUESTIONS

1.. The t.p.m of a Marko cain with three states 0,1,2 is P=

and the initial state distribution is


Find (i)P[X 2 =3] ii)P[X 3 =2, X 2 =3, X 1 =3, X 0 =2]
2. Three boys A, B, C are throwing a ball each other. A always throws the ball to B and B
always throws the ball to C, but C is just as likely to throw the ball to B as to A. S.T. the process
is Markovian. Find the transition matrix and classify the states
3. A housewife buys 3 kinds of cereals A, B, C. She never buys the same cereal in successive
weeks. If she buys cereal A, the next week she buys cereal B. However if she buys P or C the

ww
next week she is 3 times as likely to buy A as the other cereal. How often she buys each of the
cereals?
4. A man either drives a car or catches a train to go to office each day. He never goes 2 days in a

w.E
row by train but if he drives one day, then the next day he is just as likely to drive again as he is
to travel by train. Now suppose that on the first day of week, the man tossed a fair die and drove
to work if a 6 appeared. Find 1) the probability that he takes a train on the 3rd day. 2). The

asy
probability that he drives to work in the long run.

WORKED OUT EXAMPLES


En
gin
Example:1.Let X n denote the outcome of the nth toss of a fair die.
Here S = {1, 2, 3, 4, 5, 6}
T = {1, 2, 3, …}
ee
∴ (X n , n = 1, 2, 3, …} is a discrete random sequence.

rin
Example:2 Given an example of stationary random process and justify your claim.
Solution: g.n
uniformly
(0, 2π).
distribution random Variable

Since 'θ' is uniformly distributed in (0, 2π), we have


in the
et
Let us consider a random process X(t) = A as (wt + θ) where A &ω are custom and 'θ' is
interval

1
 ,0 < C < 2π
f ( θ ) = 2π
0 ,otherwise


∴ E[X(t)] = ∫ X ( t ) f ( θ ) dθ
−∞

58

Downloaded From : www.EasyEngineering.net


Downloaded From : www.EasyEngineering.net

MA6451 PROBABILITY AND RANDOM PROCESSES


1
= ∫ A ω ( ω t + θ ) 2 π dθ
0

A 2π
= sin ( ωt + θ )  0

A
Sin ( 2π + ωt ) − Sin ( ωt + 0 ) 
2π 
=

A
= [Sinωt − sin ωt ]

= 0 constant
Since E[X(t)] = a constant, the process X(t) is a stationary random process.

ww
Example:3.which are not stationary .Examine whether the Poisson process {X(t)} given by the
e −λt ( λt )
w.E
probability law P{X(t)=n] =
n
, n = 0, 1, 2, ….

Solution
We know that the mean is given by

E  X ( t )  =∑ nPn ( t )
asy
n =0
En
gin
ne −λt ( λt )
∞ n

= ∑
n =0 n

= ∑

n =1
e −λt ( λt )
n −1
ee n

rin
( λt )
g.n
∞ n

=e −λt

n =1 n −1

= −λt
e  +
 λt ( λt ) 2
 0! 1!
+ ...


et
 λt ( λt ) 2 
= ( λt ) e 1 + +
−λt
+ ... 

 1 2 
= ( λt ) e e
−λt λt

= λt , depends on t
Hence Poisson process is not a stationary process.

59

Downloaded From : www.EasyEngineering.net


Downloaded From : www.EasyEngineering.net

MA6451 PROBABILITY AND RANDOM PROCESSES

UNIT - 4
CORRELATION AND SPECTRAL DENSITY

Introduction
The power spectrum of a time series x(t) describes how the variance of the data x(t) is
distributed over the frequency components into which x(t) may be decomposed. This
distribution of the variance may be described either by a measure µ or by a statistical
cumulative distribution function S(f) = the power contributed by frequencies from 0 upto
f. Given a band of frequencies [a, b) the amount of variance contributed to x(t) by
frequencies lying within the interval [a,b) is given by S(b) - S(a). Then S is called the
spectral distribution function of x.
The spectral density at a frequency f gives the rate of variance contributed by

ww
frequencies in the immediate neighbourhood of f to the variance of x per unit frequency.

4.1 Auto Correlation of a Random Process

w.E Let X(t 1 ) and X(t 2 ) be the two given random variables. Then auto correlation is
R XX (t 1 , t 2 ) = E[X(t 1 ) X(t 2 )]
Mean Square Value

asy
Putting t 1 = t 2 = t in (1)
R XX (t, t) = E[X(t) X(t)]

En
RXX (t, t) = E[X2(t)]
Which is called the mean square value of the random process.

Auto Correlation Function gin


ee
Definition: Auto Correlation Function of the random process {X(t)} is
R XX = (τ) = E{(t) X(t+τ)}
Note: R XX (τ) = R(τ) = R X (τ)
rin
PROPERTY: 1
g.n
The mean square value of the Random process may be obtained from the auto correlation
function.
R XX(τ), by putting τ = 0.
is known as Average power of the random process {X(t)}.
PROPERTY: 2
et
R XX(τ) is an even function of τ.
R XX (τ) = R XX (-τ)

PROPERTY: 3
If the process X(t) contains a periodic component of the same period.

PROPERTY: 4
If a random process {X(t)} has no periodic components, and
E[X(t)] = X then
60

Downloaded From : www.EasyEngineering.net


Downloaded From : www.EasyEngineering.net

MA6451 PROBABILITY AND RANDOM PROCESSES

R XX ( τ ) X = lim R XX ( τ )
2
lim= (or)X
|T| →∞ |T| →∞

i.e., when τ→∞, the auto correlation function represents the square of the mean of the random
process.

PROPERTY: 5
The auto correlation function of a random process cannot have an arbitrary shape.

SOLVED PROBLEMS ON AUTO CORRELATION


Example : 1
Check whether the following function are valid auto correlation function (i) 5 sin nπ (ii)
1

ww
1 + 9τ 2

Solution:

w.E
(i) Given R XX(τ) = 5 Sin nπ
R XX (–τ) = 5 Sin n(–π) = –5 Sin nπ
R XX(τ) ≠ R XX (–τ), the given function is not an auto correlation function.

1 asy
(ii) Given R XX (τ) =
1 + 9τ 2
En
R XX (–τ) =
1
=
1 + 9 ( −τ )
2
gin
R XX ( τ )

Example : 2
ee
∴ The given function is an auto correlation function.

rin
g.n
Find the mean and variance of a stationary random process whose auto correlation
function is given by
2
R XX ( τ ) = 18 +

Solution
6 + τ2
et
2
Given R XX ( τ ) = 18 +
6 + τ2
=
X2 lim R XX ( τ )
| τ | →∞

 2 
= lim 18 +
| τ | →∞ 6 + τ2 
 
2
= 18 + lim
| τ | →∞ 6 + τ 2

61

Downloaded From : www.EasyEngineering.net


Downloaded From : www.EasyEngineering.net

MA6451 PROBABILITY AND RANDOM PROCESSES

2
= 18 +
6+
= 18 + 0
= 18
X = 18
E  X ( t )  = 18
Var {X(t)} = E[X2(t)] - {E[X(t)]}2
We know that
E  X 2 ( t )  = R XX(0)
2 55
= 18 + =
6+0 3
ww =
1

w.E
3

Example : 3

asy
Express the autocorrelation function of the process {X'(t)} in terms of the auto correlation
function of process {X(t)}

Solution
En
Consider, R XX '(t 1 , t 2 ) = E{X(t 1 )X'(t 2 )}

gin

= E  X ( t1 ) lim 
 X ( t 2 + h ) − X ( t 2 ) 


ee
n →0
  h 
 X ( t1 ) X ( t 2 + h ) − X ( t1 ) X ( t 2 ) 
= lim E 
h →0
 h rin

= lim 
 R XX ( t1 , t 2 + h ) − R X ( t1 , t 2 ) 
 g.n
⇒ R XX ' (t 1 , t 2 ) =
h →0


∂t 2

R XX ( t1 , t 2 )
h

(1)

et

Similarly R XX ' (t 1 , t 2 ) = R XX ' ( t, t 2 )
∂t1

⇒ R X ' X (t 1 , t 2 ) = R XX ( t1 , t 2 ) by (1)
∂t, ∂t 2

Auto Covariance
The auto covariance of the process {X(t)} denoted by C XX (t 1 , t 2 ) or C(t 1 , t 2 ) is defined as

62

Downloaded From : www.EasyEngineering.net


Downloaded From : www.EasyEngineering.net

MA6451 PROBABILITY AND RANDOM PROCESSES

CXX ( t1 ,=
t2 ) {  (
E  X ( t1 ) − E ( X ( t1 ) )   X t 2 − E  X ( t 2 )  
 )}
4.2 CORRELATION COEFFICIENT
C (t ,t )
ρXX ( t1 , t 2 ) =XX 1 2
Var X ( t1 ) x Var X ( t 2 )
Where C XX(t 1 , t 2 ) denotes the auto covariance.

4.3 CROSS CORRELATION


Cross correlation between the two random process {X(t)} and {Y(t)} is defined as
R XY (t 1 , t 2 ) = E[X(t 1 ) Y(t 2 )] where X(t 1 ) Y(t 2 ) are random variables.

4.4 CROSS COVARIANCE

ww {
as
Let {X(t)} and {Y(t)} be any two random process. Then the cross covariance is defined

( )}
w.E CXY ( t1 ,=
t2 ) E  X ( t1 ) − E ( Y ( t1 ) )   X t 2 − E  Y ( t 2 )  

The relation between Mean Cross Correlation and cross covariance is as follows:

C XY ( t1 , t 2 =
Definition
)
asy
R XY ( t1 , t 2 ) − E  X ( t1 ) E  Y ( t 2 )  

CXY ( t1 , t 2 ) 0, ∀ t1 , t 2En
Two random process {X(t)} and {Y(t)} are said to be uncorrelated if

Hence from the above remark we have,


gin
R XY (t 1 , t 2 ) = E[X(t 1 ) Y(t 2 )]
4.4.1 CROSS CORRELATION COEFFICIENT

ρXY ( t1 , t 2 ) =
ee
c XY ( t1 , t 2 )
Var ( X ( t1 ) ) Var ( X ( t 2 ) ) rin
g.n
4.4.1 CROSS CORRELATION AND ITS PROPERTIES
Let {X(t)} and {Y(t)} be two random. Then the cross correlation between them is also
defined as
R XY (t, t+τ) = E  X ( t ) Y ( t + τ ) 
et
= R XY (τ)

PROPERTY : 1
R XY (τ) = R YX (–τ)

PROPERTY : 2
If {X(t)} and {Y(t)} are two random process then R XY ( τ ) ≤ R XX ( 0 ) R YY ( 0 ) , where
R XX(τ) and R YY (τ) are their respective auto correlation functions.
63

Downloaded From : www.EasyEngineering.net


Downloaded From : www.EasyEngineering.net

MA6451 PROBABILITY AND RANDOM PROCESSES

PROPERTY : 3
If {X(t)} and {Y(t)} are two random process then,
R XY ( τ ) ≤ 1  R XX ( 0 ) + R YY ( 0 ) 
2

SOLVED PROBLEMS ON CROSS CORRELATION


Example:4.4.1
Two random process {X(t)} and {Y(t)} are given by
X(t) = A cos (ωt+θ), Y(t) = A sin (ωt + θ) where A and ω are constants and 'θ' is a uniform
random variable over 0 to 2π. Find the cross correlation function.

Solution

ww By def. we have
R XY (τ) = R XY (t, t+τ)

w.E
Now, R XY (t, t+τ) = E[X(t). Y(t+τ)]
= E [A cos (ωt + θ). A sin (ω (t+τ) + θ)]
{ }
= A 2 E sin ω ( t + τ ) + θ cos ( ωt + θ ) 

asy
Since 'θ' is a uniformly distributed random variable we have
f(0) =
1

, 0 ≤ θ ≤ 2π
En
gin
Now E sin {ω ( t + τ ) + θ} cos ( ωt + θ ) 

=

−∞

ee
∫ sin ( ωt + ωτ + θ ) .cos ( wt + θ ) f ( θ ) dθ
 1  rin
∫0 sin ( ωt + ω ) .cos ( ωt + θ )  2π  dθ
= t +θ

g.n
et

1
sin ( ωt + ωτ + θ ) cos ( ωt + θ ) dθ
2π ∫0
=


1

1
{sin ( ωt + ωτ + θ + ωt + θ )
= 2π 0
2

+ sin [ ωt + ωτ + θ − ωt − θ]} dθ
1

sin [ 2ωt + ωτ + 2θ] + sin ( ωτ )
=
2π ∫
0
2

64

Downloaded From : www.EasyEngineering.net


Downloaded From : www.EasyEngineering.net

MA6451 PROBABILITY AND RANDOM PROCESSES


1  cos ( 2ωt + ωτ + 2θ ) 
= − + sin ωτ ( θ ) 
4π  2 0
1  cos ( 2ωt + ωτ ) cos ( 2ωt + ωτ + 0 ) 
= − + + sin ωτ ( 2π − 0 ) 
4π  2 2 
1  cos ( 2ωt + ωτ ) cos ( 2ωt + ωτ ) 
= − + + 2π sin ωτ 
4π  2 2 
1
= [0 + 2π sin ωτ]

1
= sin ωτ (3)

ww
Substituting (3) in (1) we get
2

w.E R XY (=
t, t τ )
A2
2
sin ωτ

asy
4.5 SPECTRAL DENSITIES (POWER SPECTRAL DENSITY)
INTRODUCTION
(i) Fourier Transformation
En
(ii) Inverse Fourier Transform
gin
(iii) Properties of Auto Correlation Function
(iv) Basic Trigonometric Formula
(v) Basic Integration

4.5.1 SPECIAL REPRESENTATION


ee rin
( t ) x =
F  x = (w)

∫ x (t)e
− iωt
dt g.n
Let x(t) be a deterministic signal. The Fourier transform of x(t) is defined as

−∞
Here X(ω) is called "spectrum of x(t)".
Hence x(t) = Inverse Fourier Transform of X(ω)
et

1
= ∫ X ( ω) eiωt dω .
2π −∞

Definition
The average power P(T) of x(t) over the interval (-T, T) is given by
T
1
P (T) = ∫ x 2 ( t ) dt
2T − T

65

Downloaded From : www.EasyEngineering.net


Downloaded From : www.EasyEngineering.net

MA6451 PROBABILITY AND RANDOM PROCESSES

X T ( ω)
2

1
= ∫
2π −∞ 2T
dω (1)

Definition
The average power PXX for the random process {X(t)} is given by
T
1
PXX = lim ∫ E  X 2 ( t )  dt
T →∞ 2π
−T

E
∞  X ( ω) 2 
1  T  dω
= ∫ lim
2π −∞ T →∞ 2T
(2)

ww
4.6 POWER SPECTRAL DENSITY FUNCTION
Definition
If {X(t)} is a stationary process (either in the strict sense or wide sense) with auto

w.E
correlation function R XX(τ), then the Fourier transform of R XX (τ) is called the power spectral
density function of {X(t)} and is denoted by S XX (ω) or S(ω) or S X(ω).
S XX (ω)= Fourier Transform of R XX (τ)

=

∫ R ( τ) e
XX asy
− iωτ

Thus,
−∞

∞ En
SXX ( f ) = ∫ R ( τ) e
−∞
XX
gin
− i2 πfτ

4.6.1 WIENER KHINCHINE RELATION

( ω)

ee rin
∫ R ( τ) e
− iωτ
SXX= dτ

g.n
XX
−∞

SXX ( f ) = ∫ R ( τ) e
− i2 πfτ

−∞
XX

To find R XX(τ) if S XX(ω) or S XX(f) is given


1


et
=
R XX ( τ ) ∫ SXX ( ω) eiωτdω [inverse Fourier transform of S XX (ω)]
2π −∞

1
(or) R XX ( τ )
= ∫ SXX ( f ) e − i2 πfτdτ
2π −∞
[inverse Fourier transform of S XX(f)]

4.7 PROPERTIES OF POWER SPECTRAL DENSITY FUNCTION


Property 1:

66

Downloaded From : www.EasyEngineering.net


Downloaded From : www.EasyEngineering.net

MA6451 PROBABILITY AND RANDOM PROCESSES

The value of the spectral density function at zero frequency is equal to the total area
under the group of the auto correlation function.

SXX ( f ) = ∫ R ( τ) e
XX
− i2 πfc

−∞
Taking f = 0, we get

Sxx(0) = ∫ R ( τ ) dτ
−∞
XX

TUTORIAL QUESTIONS

1. Find the ACF of {Y(t)} = AX(t)cos (w 0 + ) where X(t) is a zero mean stationary random

ww
process with ACF
independent of X(t).
A and w 0 are constants and is uniformly distributed over (0, 2 ) and

2. Find the ACF of the periodic time function X(t) = A sinwt

w.E
3.If X(t) is a WSS process and if Y(t) = X(t + a) – X(t – a), prove that

4. If X(t) = A sin( ), where A and are constants and is a random variable, uniformly
distributed over (-
asy
), Find the A.C.F of {Y(t)} where Y(t) = X2(t).

En
5.. Let X(t) and Y(t) be defined by X(t) = Acos t + Bsin t and Y(t) = B cos t – Asin t
Where is a constant and A nd B are independent random variables both having zero mean and
varaince
processes? gin
. Find the cross correlation of X(t) and Y(t). Are X(t) and Y(t) jointly W.S.S

), where A and
ee
6. Two random processes X(t) and Y(t) are given by X(t) = A cos ( ), Y(t) = A sin(
are constants and is uniformly distributed over (0, 2 ). Find the cross
correlation of X(t) and Y(t) and verify that .
rin
7..If U(t) = X cos t + Y sin t and V(t) = Y cost + X sint t where X and Y are independent random

g.n
varables such that E(X) = 0 = E(Y), E[X2] = E[Y2] = 1, show that U(t) and V(t) are not jointly
W.S.S but they are individually stationary in the wide sense.
8. Random Prosesses X(t) and Y(t) are defined by X(t) = A cos (
where A, B and
et
), Y(t) = B cos (
are constants and is uniformly distributed over (0, 2 ). Find the cross
correlation and show that X(t) and Y(t) are jointly W.S.S
)

WORKEDOUT EXAMPLES

Example 1.Check whether the following function are valid auto correlation function (i) 5 sin nπ
1
(ii)
1 + 9τ 2
Solution:
(i) Given R XX(τ) = 5 Sin nπ
R XX (–τ) = 5 Sin n(–π) = –5 Sin nπ
67

Downloaded From : www.EasyEngineering.net


Downloaded From : www.EasyEngineering.net

MA6451 PROBABILITY AND RANDOM PROCESSES

R XX(τ) ≠ R XX (–τ), the given function is not an auto correlation function.

1
(ii) Given R XX (τ) =
1 + 9τ 2
1
R XX (–τ) = = R XX ( τ )
1 + 9 ( −τ )
2

∴ The given function is an auto correlation function.

Example : 2
Find the mean and variance of a stationary random process whose auto correlation
function is given by

ww
Solution
R XX ( τ ) = 18 +
2
6 + τ2

w.E 2
Given R XX ( τ ) = 18 +
6 + τ2
=
X2 lim R XX ( τ )
asy
| τ | →∞

 
= lim 18 +
| τ | →∞
En
6 + τ2 

2

= 18 + lim
gin
2
| τ | →∞ 6 + τ 2

= 18 +
= 18 + 0
6+
2
ee rin
X
= 18
= 18 g.n
E  X ( t ) 
Var {X(t)}
We know that
= 18
= E[X2(t)] - {E[X(t)]}2 et
E  X 2 ( t )  = R XX(0)
2 55
= 18 + =
6+0 3
1
=
3

Example : 3

68

Downloaded From : www.EasyEngineering.net


Downloaded From : www.EasyEngineering.net

MA6451 PROBABILITY AND RANDOM PROCESSES

Express the autocorrelation function of the process {X'(t)} in terms of the auto correlation
function of process {X(t)}

Solution
Consider, R XX '(t 1 , t 2 ) = E{X(t 1 )X'(t 2 )}
  X ( t 2 + h ) − X ( t 2 ) 
= E  X ( t1 ) lim  
n →0
  h 
 X ( t1 ) X ( t 2 + h ) − X ( t1 ) X ( t 2 ) 
= lim E  
h →0
 h 
 R XX ( t1 , t 2 + h ) − R X ( t1 , t 2 ) 
= lim  

ww ⇒ R XX ' (t 1 , t 2 ) =

h →0

R XX ( t1 , t 2 )
h

(1)

w.E
Similarly R XX ' (t 1 , t 2 ) =
∂t 2

∂t1
R XX ' ( t, t 2 )

⇒ R X ' X (t 1 , t 2 ) asy =

R XX ( t1 , t 2 ) by (1)

Example :4 En ∂t, ∂t 2

Two random process


gin
{X(t)} and {Y(t)} are given by
X(t) = A cos (ωt+θ), Y(t) = A sin (ωt + θ) where A and ω are constants and 'θ' is a uniform

By def. we have
R XY (τ) = R XY (t, t+τ)
ee
random variable over 0 to 2π. Find the cross correlation function.
Solution

rin
Now, R XY (t, t+τ) = E[X(t). Y(t+τ)]
= E [A cos (ωt + θ). A sin (ω (t+τ) + θ)] g.n
{ }
= A 2 E sin ω ( t + τ ) + θ cos ( ωt + θ ) 

Since 'θ' is a uniformly distributed random variable we have


et
1
f(0) = , 0 ≤ θ ≤ 2π

Now E sin {ω ( t + τ ) + θ} cos ( ωt + θ ) 

= ∫ sin ( ωt + ωτ + θ ) .cos ( wt + θ ) f ( θ ) dθ
−∞

69

Downloaded From : www.EasyEngineering.net


Downloaded From : www.EasyEngineering.net

MA6451 PROBABILITY AND RANDOM PROCESSES


 
∫ sin ( ωt + ω ) .cos ( ωt + θ )  2π  dθ
t +θ 1
=
0

1
sin ( ωt + ωτ + θ ) cos ( ωt + θ ) dθ
2π ∫0
=

∫ 2 {sin ( ωt + ωτ + θ + ωt + θ )
1 1
= 2π 0

+ sin [ ωt + ωτ + θ − ωt − θ]} dθ
1

sin [ 2ωt + ωτ + 2θ] + sin ( ωτ )
=
2π ∫ 2

ww = −
0

1  cos ( 2ωt + ωτ + 2θ ) 
+ sin ωτ ( θ ) 

w.E =
4π 

−
2

+
0
1  cos ( 2ωt + ωτ ) cos ( 2ωt + ωτ + 0 ) 
+ sin ωτ ( 2π − 0 ) 
4π 
asy 2 2
1  cos ( 2ωt + ωτ ) cos ( 2ωt + ωτ ) 

= −
4π 
En 2
+
2
+ 2π sin ωτ 

=
1
4π gin
[0 + 2π sin ωτ]
1
= sin ωτ
2
Substituting (3) in (1) we get
ee (3)

rin
R XY (=
t, t τ )
A2
2
sin ωτ
g.n
et

70

Downloaded From : www.EasyEngineering.net


Downloaded From : www.EasyEngineering.net

MA6451 PROBABILITY AND RANDOM PROCESSES

UNIT – 5

LINEAR SYSTEM WITH RANDOM INPUTS


Introduction
Mathematically a "system" is a functional relationship between the input x(t) and
y(t). We can write the relationship as
y(f) = f[x(t): –∞< + <∞]
Let x(t) represents a sample function of a random process {X(t)}. Suppose the
system produces an output or response y(f) and the ensemble of the output functions
forms a random process {Y(t)}. Then the process {Y(t)} can be considered as the output
of the system or transformation 'f' with {X(t)} as the input and the system is completely
specified by the operator "f".

ww
5.1 LINEAR TIME INVARIANT SYSTEM
Mathematically a "system" is a functional relationship between the input x(t) and output

w.E
y(t). we can write the relationship
y ( t )= f  x ( t ) : − ∞ < t < ∞
5.2 CLASSIFICATION OF SYSTEM

f a1X1 ( t ) ± a=
asy
1. Linear System: f is called a linear system, if it satisfies
2 x 2 ( t )
 a1f  X1 ( t ) ± a 2f  X 2 ( t )  
2. Time Invariant System: En
Let Y(t) = f[X(t)]
gin
If Y ( t + h ) =f  X ( t + h )  , then f is called a time invariant system or X(t) and Y(t) are said to
form a time invariant system.
3. Causal System: ee rin
Suppose the value of the output Y(t) at t = t 0 depends only on the past values of the input
X(t), t≤t 0 .
g.n
In other words, if Y ( t 0 ) =f  X ( t ) : t ≤ t 0  , then such a system is called a causal
system.
4. Memory less System: et
If the output Y(t) at a given time t = t 0 depends only on X(t 0 ) and not on any other past or
future values of X(t), then the system f is called memory less system.
5. Stable System:
A linear time invariant system is said to be stable if its response to any bounded input is
bounded.
REMARK:
i) Noted that when we write X(t) we mean X(s,t) where s ∈ S, S is the sample space. If the
system operator only on the variable t treating S as a parameter, it is called a deterministic
system.
() ()
 →  →
Input X t Output Y t
Linear System

71

Downloaded From : www.EasyEngineering.net


Downloaded From : www.EasyEngineering.net

MA6451 PROBABILITY AND RANDOM PROCESSES

h(t)
(a)
() ()
 → LTI System  →
Input X t Output Y t

h(t)
(b)
a) Shows a general single input - output linear system
b) Shows a linear time invariant system

5.3 REPRESENTATION OF SYSTEM IN THE FORM OF CONVOLUTION


Y(t) =h (t)x X(t)

wwY(t)
=
−∞
∫ h ( u ) X ( t − u ) du

w.E =

∫ h ( t − u ) X ( u ) du
asy
−∞
5.4 UNIT IMPULSE RESPONSE TO THE SYSTEM
If the input of the system is the unit impulse function, then the output or response is the
system weighting function.
Y(t) = h(t) En
Which is the system weight function.
gin
5.4.1 PROPERTIES OF LINEAR SYSTEMS WITH RANDOM INPUT
Property 1:

=Y(t)
∞ ee
If the input X(t) and its output Y(t) are related by

rin
∫ h ( u ) X ( t − u ) du , then the system is a linear time - invariant system.
Property 2:
−∞

g.n
et
If the input to a time - invariant, stable linear system is a WSS process, then the output
will also be a WSS process, i.e To show that if {X(t)} is a WSS process then the output {Y(t)} is
a WSS process.
Property 3:

If {X(t)} is a WSS process and if Y(t) = ∫ h ( u ) X ( t − u ) du ,
−∞
then

R XY (=
τ ) R XX ( τ ) x h ( τ )
Property 4 :

If {(X(t)} is a WSS process and if Y(t) = ∫ h ( u ) X ( t − u ) du ,
−∞
then

( τ ) R XY ( τ ) x h ( −τ )
R YY=
72

Downloaded From : www.EasyEngineering.net


Downloaded From : www.EasyEngineering.net

MA6451 PROBABILITY AND RANDOM PROCESSES

Property 5:

If {X(t)} is a WSS process and if Y(t) = ∫ h ( u ) X ( t − u ) du ,
−∞
then

( τ ) R XX ( τ ) x h ( τ ) x h ( −τ )
R YY=

Property 6:

If {X(t)} is a WSS process if Y(t) =
−∞
∫ h ( u ) X ( t − u ) du , then S ( ω=)
XY SXX ( ω) H ( ω)

. Property 7:

If {X(t)} is a WSS process and if Y(t) = ∫ h ( u ) X ( t − u ) du , then

ww
SYY ( ω=
) SXX ( ω) H ( ω)
2
−∞

w.E
Note:
Instead of taking R XY ( τ )= E  X ( t ) Y ( t + τ )  in properties (3), (4) & (5), if we start

( τ ) R XY ( τ ) xh ( −τ )
a) R XY= asy
with R XY ( τ )= E  X ( t − τ ) Y ( t )  , then the above property can also stated as

( τ ) R XY ( τ ) xh ( −τ )
b) R YY=
En
gin
c) R YY ( τ ) = R XX ( τ ) x h ( −τ ) x h ( τ )
REMARK :
(i) We have written H ( ω) H * ( ω)=
H ( ω) = F  h ( τ ) 
ee
H ( ω) because
2

rin
H * ( ω)= F  h ( −τ ) 

(
=  F h ( τ ) ) g.n
= H ( ω)
(ii) Equation (c) gives a relationship between the spectral densities of the input and output
et
process in the system.
(iii) System transfer function:
We call H ( ω
= ) F{h ( τ )} as the power transfer function or system transfer function.
SOLVED PROBLEMS ON AUTO CROSS CORRELATION FUNCTIONS OF INPUT
AND OUTPUT
Example :5.4.1
Find the power spectral density of the random telegraph signal.
Solution
We know, the auto correlation of the telegraph signal process X(y) is
73

Downloaded From : www.EasyEngineering.net


Downloaded From : www.EasyEngineering.net

MA6451 PROBABILITY AND RANDOM PROCESSES

R XX ( τ ) =e
−2 λ τ

∴ The power spectral density is


( ω)
SXX= ∫ R ( τ) e
XX
− iωτ

−∞
0 ∞

∫e dτ + ∫ e −2 λτe − iωτdτ
2 λτ − iωτ
= e
−∞ 0

 τ = −τ when τ < 0

 τ = −τ when τ > 0

0 ∞
( 2 λ−iω)τ

ww = ∫e
−∞

 e( 2 λ−iω)τ 
dτ + ∫ e −2 λτe − iωτdτ
0
0

 e −( 2 λ+iω)τ 

w.E =   +
 ( 2λ − iω)  ∞  − ( 2λ + iω)  0

=
1
( 2λ − iω) asy
e0 − e −∞  −
1
( 2λ + iω)
e −∞ − e −0 

=
1
( 2λ − iω)
(1 − 0 ) − En
1
( 2λ + i ω )
( 0 − 1)

=
1
+
1 gin
e −∞ − e −0 

=
( 2λ − i ω ) ( 2 λ + i ω )
1
( 2λ − iω)
(1 − 0 ) +
ee rin
=
2λ + iω + 2λ − iω
( 2λ − iω)( 2λ + iω)
g.n
SXX ( ω) = 2

4λ + ω2
et
Example : 5.4.2
A linear time invariant system has a impulse response h ( t ) = e −βt U ( t ) . Find the power
spectral density of the output Y(t) corresponding to the input X(t).
Solution:
Given X(t) - Input
Y(t) - output
S YY (ω) - |H(ω)|2 S XX(ω)

74

Downloaded From : www.EasyEngineering.net


Downloaded From : www.EasyEngineering.net

MA6451 PROBABILITY AND RANDOM PROCESSES


Now H ( ω) = h ( t ) e − iωt dt

−∞
0 ∞

∫ h (t)e dt + ∫ e −βt e − iωt dt


− iωt
=
−∞ 0

= 0+ e ( ) dt∫
− β+ iω t

0

 e −(β+iω)t 
=  
 − ( β + iω )  0
1
(
e −∞ − e −0 )
ww ( ) =
− ( β + iω )
1

w.E =
− ( β + iω )
1
e −∞ − e −0

asy =
β + iω
1
|H(ω)|
En =
β + iω

gin
=
1
β2 + ω2

∴ SYY ( ω)
=
1
ee
β + ω2
2
SXX ( ω)
rin
TUTORIAL QUESTIONS
g.n
1. State and Prove Power spectral density of system response theorem.
et
2. Suppose that X(t) is the input to an LTI system impulse response h1 (t) and that Y(t) is the
input to another LTI system with impulse response h2 (t). It is assumed that X(t) and Y(t) are
jointly wide sense stationary. Let V(t) and Z(t) denote that random processes at the respective
system outputs. Find the cross correlation of X(t) and Y(t).
3. The input to the RC filter is a white noise process with ACF . If the
frequency response find the auto correlation and the mean square value of the
output process Y(t).
4. A random process X(t0 having ACF , where P and
are real positive
λ e − λt , t > 0
constants, is applied to the input of the system with impulse response h(t) =  where
0, t < 0
75

Downloaded From : www.EasyEngineering.net


Downloaded From : www.EasyEngineering.net

MA6451 PROBABILITY AND RANDOM PROCESSES

λ is a real positive constant. Find the ACF of the network’s response Y(t). Find the cross
correlation .

WORKEDOUT EXAMPLES
Example: 1
Find the power spectral density of the random telegraph signal.
Solution
We know, the auto correlation of the telegraph signal process X(y) is
R XX ( τ ) =e
−2 λ τ

∴ The power spectral density is


( ω)
SXX= ∫ R ( τ) e
− iωτ

ww −∞
0
XX

∫ e e dτ + ∫ e e dτ
2 λτ − iωτ −2 λτ − iωτ

w.E =
−∞ 0

 τ = −τ when τ < 0

asy 
 τ = −τ when τ > 0

=
0

∫e
( 2 λ−iω)τ
En

dτ + ∫ e −2 λτe − iωτdτ

 e
−∞

( 2 λ−iω)τ

0
0

gin
 e −( 2 λ+iω)τ 

= 

=
 ( 2λ
1
− i ω )
 +
∞  − ( 2λ

e0 − e −∞  −
+ i ω
ee
1
)

0
e −∞ − e −0  rin
( 2λ − iω) ( 2λ + iω)
1
(1 − 0 ) −
1
( 0 − 1)
g.n
=

=
( 2λ − iω)
1
+
1
( 2λ + i ω )
e −∞ − e −0 
et
( 2λ − i ω ) ( 2 λ + i ω )
1
= (1 − 0 ) +
( 2λ − iω)
2λ + iω + 2λ − iω
=
( 2λ − iω)( 2λ + iω)

SXX ( ω) = 2
4λ + ω2

76

Downloaded From : www.EasyEngineering.net


Downloaded From : www.EasyEngineering.net

Probability & Random Process


Formulas
UNIT-I (RANDOM VARIABLES)

1) Discrete random variable:


A random variable whose set of possible values is either finite or countably 
infinite is called discrete random variable. 

ww   Eg: (i) Let X represent the sum of the numbers on the 2 dice, when two 
dice are thrown.  In this case the random variable X takes the values 2, 3, 4, 5, 6, 

w.E
7, 8, 9, 10, 11 and 12.  So X is a discrete random variable. 
        (ii) Number of transmitted bits received in error. 
2) Continuous random variable:

asy
A random variable X is said to be continuous if it takes all possible values 
between certain limits. 

En
  Eg: The length of time during which a vacuum tube installed in a circuit 
functions is a continuous random variable, number of scratches on a surface, 

gin
proportion of defective parts among 1000 tested, number of transmitted in 
error. 

ee
3)
Sl.No. Discrete random variable Continuous random variable



p( x i ) = 1  

f ( x )dx = 1   rin

i =−∞

F ( x) = P [ X ≤ x]  
−∞

F ( x) = P [ X ≤ x] =
g.n x


et
f ( x )dx  
−∞

3  Mean = E [ X ] = ∑ xi p( xi )   ∞

i Mean = E [ X ] = ∫ xf ( x )dx
−∞

4  E  X  = ∑ x p( xi )  
2 2 ∞

E  X 2  = ∫x
i 2
i f ( x )dx
−∞

Var ( X ) = E ( X 2 ) −  E ( X )    Var ( X ) = E ( X 2 ) −  E ( X ) 
2 2

6  Moment =  E  X r  = ∑ xir pi   ∞

Moment =  E  X r  = ∫x
r
i f ( x )dx
−∞

7  M.G.F  M.G.F 

Downloaded From : www.EasyEngineering.net


Downloaded From : www.EasyEngineering.net

M X ( t ) = E  e tX  = ∑ e tx p( x )   ∞

x M X ( t ) = E  e tX  = ∫e
tx
f ( x )dx  
−∞

4) E ( aX + b ) = aE ( X ) + b
5) Var ( aX + b ) = a 2 Var ( X )
6) Var ( aX ± bY ) = a 2 Var ( X ) + b 2Var (Y )
7) Standard Deviation = Var ( X )
8) f ( x ) = F ′( x )
9) p( X > a ) = 1 − p( X ≤ a )
p( A ∩ B)
ww
10) p ( A / B ) =
p( B)
, p( B) ≠ 0

w.E
11) If A and B are independent, then p ( A ∩ B ) = p ( A ) ⋅ p ( B ) . 

12) 1st Moment about origin =  E [ X ]  =   M X ′ ( t )     (Mean)


  t =0

asy
2nd Moment about origin =  E  X 2   =   M X ′′ ( t )   
  t =0
t r

En
The co-efficient of   =  E  X r    (rth Moment about the origin)

gin
r!
13) Limitation of M.G.F:
i) A random variable X may have no moments although its m.g.f exists.

ee
ii) A random variable X can have its m.g.f and some or all moments, yet the 
m.g.f does not generate the moments.
iii) A random variable X can have all or some moments, but m.g.f does not 
rin
exist except perhaps at one point.
14) Properties of M.G.F:
i) If Y = aX + b, then  MY ( t ) = e bt M X ( at ) .
g.n
ii)
iii)
M cX ( t ) = M X ( ct ) , where c is constant.
If X and Y are two independent random variables then 
et
M X +Y ( t ) = M X ( t ) ⋅ M Y ( t ) .
15) P.D.F, M.G.F, Mean and Variance of all the distributions:
Sl. Distributio P.D.F ( P ( X = x ) ) M.G.F Mean Variance
No. n
nc x p x q n− x   ( q + pe ) np   npq  
1  Binomial  n
t

 
2  Poisson  −λ
e λ x
e
(
λ e t −1 )  λ  λ 
 
x!
3  Geometric  
q x −1 p  (or)  q x p   pe t 1 q
     
1 − qe t p p2

Downloaded From : www.EasyEngineering.net


Downloaded From : www.EasyEngineering.net

4  Uniform  
 1 e bt − e at a + b (b − a )2
 , a< x<b  
f ( x) =  b − a   ( b − a )t 2 12
 0, otherwise    
5  Exponential  
 λ e − λ x , x > 0, λ > 0 λ 1 1
f ( x) =         
 0, otherwise λ−t λ λ2
6  Gamma 
e − x x λ −1 1 λ  λ 
f ( x) = , 0 < x < ∞, λ > 0    
Γ(λ ) (1 − t )λ
7  Normal   −1  x − µ 
2
µt +
t 2σ 2 µ  σ2 
1 2  σ 
f ( x) = e      e 2
 
σ 2π

ww
 
16) Memoryless property of exponential distribution 
P ( X > S + t / X > S ) = P ( X > t ) . 

w.E
17) Function of random variable:  fY ( y ) = f X ( x )
dx
dy

UNIT-II (RANDOM VARIABLES) asy


1) ∑∑ p En
= 1   (Discrete random variable) 

∞ ∞
i j
ij

gin
∫∫
−∞ −∞ ee
f ( x , y )dxdy = 1   (Continuous random variable) 

rin P ( x, y )
2) Conditional probability function X given Y  P { X = xi / Y = yi } =
g.n P( y)

Conditional probability function Y given X  P {Y = yi / X = xi } =

P ( X < a,Y < b )


et P ( x, y )
P( x)

P { X < a / Y < b} =  
P (Y < b )

f ( x, y)
3) Conditional density function of X given Y,  f ( x / y) = . 
f ( y)

f ( x, y)
Conditional density function of Y given X,  f ( y / x) = . 
f ( x)
4) If X and Y are independent random variables then 
f ( x , y ) = f ( x ). f ( y )      (for continuous random variable) 

Downloaded From : www.EasyEngineering.net


Downloaded From : www.EasyEngineering.net

P ( X = x , Y = y ) = P ( X = x ) . P (Y = y ) (for discrete random variable) 
d b

5) Joint probability density function P ( a ≤ X ≤ b, c ≤ Y ≤ d ) = ∫ ∫ f ( x , y )dxdy . 
c a

b a

P ( X < a , Y < b ) = ∫ ∫ f ( x , y )dxdy  


0 0

6) Marginal density function of X,  f ( x ) = f X ( x ) = ∫
−∞
f ( x , y )dy  

Marginal density function of Y,  f ( y ) = fY ( y ) = ∫
ww
7) P ( X + Y ≥ 1) = 1 − P ( X + Y < 1)
−∞
f ( x , y )dx  

w.E
8) Correlation co – efficient (Discrete):  ρ ( x , y ) =
Cov ( X , Y )
σ Xσ Y

1

asy
XY − XY ,    σ X =
1
∑ X 2 − X 2 ,    σ Y =
1

Cov ( X , Y ) =
n
En n n
Y 2 −Y 2  

gin
9) Correlation co – efficient (Continuous):  ρ ( x , y ) =
Cov ( X , Y )
σ Xσ Y

ee
Cov ( X , Y ) = E ( X , Y ) − E ( X ) E (Y ) ,    σ X = Var ( X ) ,    σ Y = Var (Y )

10) If X and Y are uncorrelated random variables, then Cov ( X , Y ) = 0 . rin


11) E ( X ) =
∞ ∞ ∞ ∞
g.n
∫ xf ( x )dx ,    E (Y ) = ∫ yf ( y )dy ,   E ( X ,Y ) = ∫ ∫ xyf ( x , y )dxdy .
−∞ −∞

12) Regression for Discrete random variable:


−∞ −∞

et
Regression line X on Y is  x − x = bxy ( y − y ) ,   bxy =
∑( x − x)( y − y)
∑( y − y)
2

Regression line Y on X is  y − y = b yx ( x − x ) ,  byx =
∑( x − x)( y − y)
∑( x − x)
2

Correlation through the regression,   ρ = ± bXY .bYX       Note:  ρ ( x , y ) = r ( x , y )  

Downloaded From : www.EasyEngineering.net


Downloaded From : www.EasyEngineering.net

13) Regression for Continuous random variable:


σx
Regression line X on Y is  x − E ( x ) = bxy ( y − E ( y ) ) ,   bxy = r
σy

σy
Regression line Y on X is  y − E ( y ) = b yx ( x − E ( x ) ) ,   b yx = r  
σx

Regression curve X on Y is   x = E ( x / y) = ∫ x f ( x / y ) dx    
−∞

Regression curve Y on X is   y = E ( y / x) = ∫ y f ( y / x ) dy    

ww
14) Transformation Random Variables:
−∞

w.E
  fY ( y ) = f X ( x )
dx
dy
      (One dimensional random variable) 

asy ∂u ∂u

En
fUV ( u, v ) = f XY ( x , y )
∂x
∂v
∂y
∂v
  (Two dimensional random variable)

gin
∂x ∂y

ee
15) Central limit theorem (Liapounoff’s form)

rin
If X1, X2, …Xn be a sequence of independent R.Vs with E[Xi] = µi and Var(Xi) = σi2, i 
= 1,2,…n and if  Sn =  X1 +  X2 + … + Xn then under certain general conditions, Sn 
n
g.n
follows a normal distribution with mean  µ = ∑ µi and variance  σ 2 = ∑ σ i2  as 
n

n→∞.
i =1

et i =1

16) Central limit theorem (Lindberg – Levy’s form)


If X1, X2, …Xn be a sequence of independent identically distributed R.Vs with E[Xi] 
= µi and Var(Xi) = σi2, i = 1,2,…n and if  Sn =  X1 +  X2 + … + Xn then under certain 
general conditions, Sn follows a normal distribution with mean  nµ and variance 

nσ 2  as  n → ∞ . 

Sn − nµ X −µ
Note:  z =   ( for n variables),             z =  ( for single variables) 
σ n σ
n

Downloaded From : www.EasyEngineering.net


Downloaded From : www.EasyEngineering.net

UNIT-III (MARKOV PROCESSES AND MARKOV CHAINS)

1) Random Process:
A random process is a collection of random variables {X(s,t)} that are 
functions of a real variable, namely time ‘t’ where s Є S and t Є T. 
 
2) Classification of Random Processes:
We can classify the random process according to the characteristics of time t 
and the random variable X.  We shall consider only four cases based on t and X 
having values in the ranges -∞< t <∞ and   -∞ < x < ∞. 

ww
 
Continuous random process 

w.E Continuous random sequence 

Discrete random process 

asy
Discrete random sequence 

Continuous random process:


En
Example:  gin
If X and t are continuous, then we call X(t) , a Continuous Random Process. 
If X(t) represents the maximum temperature at a place in the 

Continuous Random Sequence: ee


interval (0,t), {X(t)} is a Continuous Random Process. 

rin
A random process for which X is continuous but time takes only discrete values is 
called a Continuous Random Sequence.  
g.n
Example:  If Xn represents the temperature at the end of the nth hour of a day, then 
{Xn, 1≤n≤24} is a Continuous Random Sequence. 
Discrete Random Process: et
If X assumes only discrete values and t is continuous, then we call such random 
process {X(t)} as Discrete Random Process. 
Example:  If X(t) represents the number of telephone calls received in the interval 
(0,t) the {X(t)} is a discrete random process since S = {0,1,2,3, . . . } 
Discrete Random Sequence:
A random process in which both the random variable and time are discrete is called 
Discrete Random Sequence. 
Example:  If Xn represents the outcome of the nth toss of a fair die, the {Xn : n≥1} is a 
discrete random sequence.  Since T = {1,2,3, . . . } and S = {1,2,3,4,5,6} 

Downloaded From : www.EasyEngineering.net


Downloaded From : www.EasyEngineering.net

3) Condition for Stationary Process:  E [ X ( t )] = Constant , Var [ X ( t )] = constant .


If the process is not stationary then it is called evolutionary. 
 
4) Wide Sense Stationary (or) Weak Sense Stationary (or) Covariance Stationary:
A random process is said to be WSS or Covariance Stationary if it satisfies the 
following conditions. 
i) The mean of the process is constant (i.e)  E ( X ( t ) ) = constant . 
ii) Auto correlation function depends only on  τ (i.e)
RXX (τ ) = E [ X ( t ). X ( t + τ )]  
5) Time average:
T
1
The time average of a random process  { X ( t )} is defined as X T =
ww 1
2T
T

If the interval is  ( 0,T ) , then the time average is  X T = ∫ X ( t ) dt . 


∫ X (t ) dt . 
−T

6)
w.EErgodic Process:
T 0

asy
A random process  { X ( t )} is called ergodic if all its ensemble averages are 
interchangeable with the corresponding time average X T . 
7) Mean ergodic:
En
Let  { X ( t )} be a random process with mean  E [ X ( t )] = µ  and time average X T , 

gin
then  { X ( t )} is said to be mean ergodic if  X T → µ  as  T → ∞  (i.e) 
E [ X ( t )] = Lt X T . 
T →∞
ee
Note:  Lt var ( X T ) = 0  (by mean ergodic theorem) 
T →∞
rin
8) Correlation ergodic process:
g.n
The stationary process  { X ( t )} is said to be correlation ergodic if the process 

9)
Where  YT is the time average of  Y ( t ) . 
Auto covariance function:
et
{Y ( t )} is mean ergodic where  Y ( t ) = X ( t ) X ( t + τ ) . (i.e)  E (Y ( t ) ) = TLt
→∞
YT . 

C XX (τ ) = RXX (τ ) − E ( X ( t ) ) E ( X ( t + τ ) )  
10) Mean and variance of time average:
T
1
Mean:    E  X T  = ∫ E [ X ( t )] dt  
T 0
2T
1
Variance:  Var  X T  = ∫ RXX (τ )C XX (τ ) dτ  
2T −2 T

Downloaded From : www.EasyEngineering.net


Downloaded From : www.EasyEngineering.net

11) Markov process:


A random process in which the future value depends only on the present value 
and not on the past values, is called a markov process. It is symbolically 
represented by  P  X ( t n +1 ) ≤ xn +1 / X ( t n ) = xn , X ( t n −1 ) = xn −1 ... X ( t 0 ) = x0   
= P  X ( t n +1 ) ≤ xn +1 / X ( t n ) = xn   
    Where  t 0 ≤ t1 ≤ t 2 ≤ ... ≤ t n ≤ t n +1  
12) Markov Chain:
If for all  n , 
P  X n = an / X n −1 = an −1 , X n − 2 = an − 2 , ... X 0 = a0  = P  X n = an / X n −1 = an −1     
then the process  { X n } ,  n = 0,1, 2, ...  is called the markov chain. Where 

ww a0 , a1 , a2 , ...an , ...  are called the states of the markov chain. 


13) Transition Probability Matrix (tpm):

w.E
When the Markov Chain is homogenous, the one step transition probability is 
denoted by Pij.  The matrix P = {Pij} is called transition probability matrix. 
14) Chapman – Kolmogorov theorem:

asy
If ‘P’ is the tpm of a homogeneous Markov chain, then the n – step tpm P(n) is 
n
equal to Pn.  (i.e)  Pij( n ) =  Pij  . 

En
15) Markov Chain property: If  Π = ( Π 1 , Π 2 , Π 3 ) , then  ΠP = Π  and 
Π 1 + Π 2 + Π 3 = 1 . 
16) Poisson process: gin
ee
If  X ( t ) represents the number of occurrences of a certain event in  (0, t ) ,then 
the discrete random process  { X ( t )} is called the Poisson process, provided the 
following postulates are satisfied. 
rin
 
(i) P [1 occurrence in ( t , t + ∆t )] = λ∆t + O ( ∆t )   g.n
(ii)
(iii)
(iv)
P [ 0 occurrence in ( t , t + ∆t )] = 1 − λ ∆t + O ( ∆t )  
P [ 2 or more occurrences in ( t , t + ∆t )] = O ( ∆t )  
X ( t )  is independent of the number of occurrences of the event in any 
et
interval. 
e−λt ( λ t )
x

17) Probability law of Poisson process:  P { X ( t ) = x} = , x = 0,1, 2, ...∞  


x!
Mean E [ X ( t )] = λ t ,   E  X 2 ( t )  = λ 2 t 2 + λ t , Var [ X ( t )] = λ t . 

UNIT-IV (CORRELATION AND SPECTRAL DENSITY)

RXX (τ )  - Auto correlation function 

Downloaded From : www.EasyEngineering.net


Downloaded From : www.EasyEngineering.net

S XX ( ω )  - Power spectral density (or) Spectral density 

RXY (τ )  - Cross correlation function 

S XY ( ω )  - Cross power spectral density 

1) Auto correlation to Power spectral density (spectral density):


  S XX ( ω ) = ∫ R (τ ) e
XX
− iωτ
dτ  
−∞

2) Power spectral density to Auto correlation:

ww

1
RXX (τ ) = ∫ S (ω ) e dω  
ωτ i
   
2π −∞
XX

w.E
3) Condition for X ( t ) and X ( t + τ ) are uncorrelated random process is

C XX (τ ) = RXX (τ ) − E [ X ( t )] E [ X ( t + τ )] = 0  

4) asy
Cross power spectrum to Cross correlation:

RXY (τ ) =
1
2π En

∫ S ( ω ) e dω  
ωτ i

gin
XY
−∞

5) General formula:

i) ∫ e cos bx dx =
ax
ee e ax
a 2 + b2
( a cos bx + b sin bx )  
rin
ii) ∫ e ax
sin bx dx =
e ax
a 2 + b2
( a sin bx − b cos bx )  
g.n
iii) 2 

a  a2
x + ax =  x +  −
2 4
 
2

et
e iθ − e − iθ
iv) sin θ =  
2i
e iθ + e − iθ
v) cos θ =  
2
 
 
 

Downloaded From : www.EasyEngineering.net


Downloaded From : www.EasyEngineering.net

UNIT-V (LINEAR SYSTEMS WITH RANDOM INPUTS)

1) Linear system:
f is called a linear system if it satisfies 

f a1 X 1 ( t ) ± a2 X 2 ( t ) = a1 f ( X 1 ( t ) ) ± a2 f ( X 2 ( t ) )  

2) Time – invariant system:


Let  Y ( t ) = f ( X ( t ) ) . If  Y ( t + h) = f ( X ( t + h) )  then  f  is called a time – 

invariant system. 

ww 3) Relation between input X ( t ) and output Y ( t ) :


w.E Y (t ) = ∫ h(u) X (t − u) du  
−∞

Where  h( u)  system weighting function. 

asy
4) Relation between power spectrum of X ( t ) and output Y ( t ) :

En
SYY (ω ) = S XX (ω ) H (ω )  
2

gin
If  H (ω )  is not given use the following formula  H (ω ) =

∫e
− jω t
h( t ) dt   

5) Contour integral:

e imx π −ma
ee rin
−∞

∫−∞ a 2 + x 2 = a e     (One of the result) 
g.n
 1  e
6) F −1  2 2
a + ω 
=
−aτ

2a
 
τ
  (from the Fourier transform)  et

Downloaded From : www.EasyEngineering.net


Downloaded From : www.EasyEngineering.net

WWW.VIDYARTHIPLUS.COM

UNIT I: PROBABILITY AND RANDOM VARIABLES

PART B QUESTIONS

1. A random variable X has the following probability distribution

X 0 1 2 3 4 5 6 7
P(X) 0 K 2k 2k 3k k2 2k2 7k2+k
Find (i) The value of k, (ii) P[ 1.5 < X < 4.5 / X >2 ] and (iii) The smallest value of λ
for which P(X ≤ λ) < (1/2).
2. A bag contains 5 balls and its not known how many of them are white. Two balls
are drawn at random from the bag and they are noted to be white. What is the
chance that the balls in the bag all are white.

ww 1 x
3. Let the random variable X have the PDF f(x) = e 2 , x >0 Find the moment
2

w.E
generating function, mean and variance.
4. A die is tossed until 6 appear. What is the probability that it must tossed
more than 4 times.

asy
5. A man draws 3 balls from an urn containing 5 white and 7 black balls. He
gets Rs. 10 for each white ball and Rs 5 for each black ball. Find his
expectation.
En
6. In a certain binary communication channel, owing to noise, the probability

gin
that a transmitted zero is received as zero is 0.95 and the probability that a
transmitted one is received as one is 0.9. If the probability that a zero is
transmitted is 0.4, find the probability that (i) a one is received (ii) a one was

ee
transmitted given that one was received

rin
7. Find the MGF and rth moment for the distribution whose PDF is f(x) = k e –x ,
x >0. Find also standard deviation.

g.n
8. The first bag contains 3 white balls, 2 red balls and 4 black balls. Second
bag contains 2 white, 3 red and 5 black balls and third bag contains 3

et
white, 4 red and 2 black balls. One bag is chosen at random and from it 3
balls are drawn. Out of 3 balls, 2 balls are white and 1 is red. What are the
probabilities that they were taken from first bag, second bag and third bag.
9. A random variable X has the PDF f(x) = 2x, 0 < x < 1 find (i) P (X < ½) (ii) P (
¼ < X < ½) (iii) P ( X > ¾ / X > ½ )
10. If the density function of a continuous random variable X is given by
ax 0≤x≤1
a 1≤x≤2
f(x) = 3a – ax 2≤x≤3
0 otherwise

(1) Find a (2) Find the cdf of X

11. If the the moments of a random variable X are defined by E ( X r ) = 0.6,


r = 1,2,.. Show that P (X =0 ) = 0.4 P ( X = 1) = 0.6, P ( X  2 ) = 0.

Downloaded From : www.EasyEngineering.net


Downloaded From : www.EasyEngineering.net

WWW.VIDYARTHIPLUS.COM

12. In a continuous distribution, the probability density is given by f(x) = kx (2


– x) 0 < x < 2. Find k, mean , varilance and the distribution function.

13. The cumulative distribution function of a random variable X is given by


0, x<0
x2, 0≤x≤½
3
F(x) = 1 - (3  x) 2 ½≤x≤3
25
1 x 3
Find the pdf of X and evaluate P ( |X| ≤ 1 ) using both the pdf and cdf

14. Find the moment generating function of the geometric random variable

ww with the pdf f(x) = p q x-1, x = 1,2,3.. and hence find its mean and variance.

w.E
15. A box contains 5 red and 4 white balls. A ball from the box is taken our at
random and kept outside. If once again a ball is drawn from the box, what
is the probability that the drawn ball is red?

5
1 3  asy
16. A discrete random variable X has moment generating function

M X(t) =   et  Find E(x), Var(X) and P (X=2)


4 4 
En
17. The pdf of the samples of the amplitude of speech wave foem is found to

gin
decay exponentially at rate , so the following pdf is proposed f(x) =
Ce | x| , -  < X < . Find C, E(x)

18.
ee rin
Find the MGF of a binomial distribution and hence find the mean and

g.n
variance.
. Find the recurrence relation of central moments for a binomial distribution.

19. The number of monthly breakdowns of a computer is a RV having a


et
poisson distribution with mean equal to 1.8. Find the probability that this
computer will function for a month (a) without a breakdown, (b) Wish only one
breakdown, (c) Wish at least one break down.
20. Find MGF and hence find mean and variance form of binomial distribution.

21. State and prove additive property of poisson random variable.

22. If X and Y are two independent poisson random variable, then show that
probability distribution of X given X+Y follows binomial distribution.
23. Find MGF and hence find mean and variance of a geometric distribution.

24. State and prove memory less property of a Geometric Distribution.

25. Find the mean and variance of a uniform distribution.

Downloaded From : www.EasyEngineering.net


Downloaded From : www.EasyEngineering.net

WWW.VIDYARTHIPLUS.COM

UNIT- II TWO DIMENSIONAL RANDOM VARIABLES


Part- B
1. If f (x, y) = x+y , 0< x <1, 0< y < 1
0 , Otherwise
Compute the correlation cp- efficient between X and Y.

2. The joint p.d.f of a two dimensional randm variable (X, Y) is given by f(x, y) = (8
/9) xy, 1 ≤ x ≤ y ≤ 2 find the marginal density unctions of X and Y. Find also the
conditional density function of Y / X =x, and X / Y = y.

3. The joint probability density function of X and Y is given by f(x, y) = (x + y) /3, 0


≤ x ≤1 & 0<y <2 obtain the egression of Y on X and of X on Y.

ww
4. If the joint p.d.f. of two random variable is given by f(x1, x2) = 6 e
-2x1 – 3 x2, x
> 0, x2 >0. Find the probability that the first random variable will take on a value
1

w.E
between 1 and 2 and the second random variable will take on avalue between 2
and 3. \also find the probability that the first random variable will take on a value
less than 2 and the second random variable will take on a value greater than 2.

asy
5. If two random variable have hoing p.d.f. f(x1, x2) = ( 2/ 3) (x1+ 2x2) 0< x1 <1,
0< x2 < 1
En
6. Find the value of k, if f(x,y) = k xy for 0 < x,y < 1 is to be a joint density function.

gin
Find P(X + Y < 1 ) . Are X and Y independent.
7. If two random variable has joint p.d.f. f(x, y) = (6/5) (x +y2), 0 < x < 1 , 0< y

ee
<1.Find P(0.2 < X < 0.5) and P( 0.4 < Y < 0.6)

rin
8. Two random variable X and Y have p.d.f f(x, y) = x2 + ( xy / 3), 0 ≤ x ≤1,
0≤ y ≤ 2. Prove that X and Y are not independent. Find the conditional density
function
g.n

 x2  y 2 
9. X and Y are 2 random variable joint p.d.f. f(x, y) = 4xy
the p. d. f. of x 2 +y2 .
e
et , x ,y ≥ 0, find

10. Two random variable X and Y have joint f(x y) 2 – x –y, 0< x <1, 0< y < 1. Find
the Marginal probability density function of X and Y. Also find the conditional
density unction and covariance between X and Y.

11. Let X and Y be two random variables each taking three values –1, 0 and
1 and having the joint p.d.f.
X
Y
-1 0 1 Prove that X and Y have different
-1 0 0.1 0.1 expections. Also Prove that X and Y are
0 0.2 0.2 0.2 uncorrelated and find Var X and Var Y
1 0 0.1 0.1

Downloaded From : www.EasyEngineering.net


Downloaded From : www.EasyEngineering.net

WWW.VIDYARTHIPLUS.COM

12. 20 dice are thrown. Find the approximate probability tat the sum obtained is
between 65 and 75 using central limit theorem.

13. Examine whether the variables X and Y are independent whose joint density is
–xy – x, 0< x , y < ∞
f(x ,y) = x e
.
14. Let X and Y be independent standard normal random variables. Find the pdf of
z =X / Y.

15. Let X and Y be independent uniform random variables over (0,1) . Find the

ww PDF of Z = X + Y

UNIT-III CLASSIFICATION OF RANDOM PROCESS

w.E PART B

asy
1. The process { X(t) } whose proabability distribution is given by
 at 
n 1

, n  1, 2...
P [ X(t) = n] =
 
1  at
 
at
Engi
n 1

=
1  at nee
,n  0
Show that it is not stationary

rin
g.n
2. A raining process is considered as a two state Markov chain. If it rains, it is
considered to be in state 0 and if it does not rain, the chain is in state 1. the
 0.6 0.4 
transitioin probability of the markov chain is defined as P  
 0.2 0.8  et
 . Find the
probability of the Markov chain is defined as today assuming that it is raining
today. Find also the unconditional probability that it will rain after three days with
the initial probabilities of state 0 and state 1 as 0.4 and 0.6 respectively.
3. Let X(t) be a Poisson process with arrival rate . Find E {( X (t) – X (s)2 } for t > s.
4. Let { Xn ; n = 1,2..} be a Markov chain on the space S = { 1,2,3} with on step
 0 1 0 
 
transition probability matrix P  1/ 2 0 1/ 2  (1) Sketch transition diagram (2) Is
 1 0 0 
 
the chain irreducible? Explain. (3) Is the chain Ergodic? Explain
5. Consider a random process X(t) defined by X(t) = U cost + (V+1) sint, where U
and V are independent random variables for which E (U ) = E(V) = 0 ; E (U 2) = E

WWW.VIDYARTHIPLUS.COM

Downloaded From : www.EasyEngineering.net


Downloaded From : www.EasyEngineering.net

WWW.VIDYARTHIPLUS.COM

( V2 ) = 1 (1) Find the auto covariance function of X (t) (2) IS X (t) wide sense
stationary? Explain your answer.
6. Discuss the pure birth process and hence obtain its probabilities, mean and
variance.
7. At the receiver of an AM radio, the received signal contains a cosine carrier
signal at the carrier frequency  with a random phase  that is uniform distributed
over ( 0,2). The received carrier signal is X (t) = A cos(t +  ). Show that the
process is second order stationary
8. Assume that a computer system is in any one of the three states busy, idle and
under repair respectively denoted by 0,1,2. observing its state at 2 pm each day,
 0.6 0.2 0.2 
 
we get the transition probability matrix as P   0.1 0.8 0.1  . Find out the 3rd
 0.6 0 0.4 
ww  
step transition probability matrix. Determine the limiting probabilities.

w.E
9. Given a random variable  with density f () and another random variable 
uniformly distributed in (-, ) and independent of  and X (t) = a cos (t + ),

asy
Prove that { X (t)} is a WSS Process.

En
10. A man either drives a car or catches a train to go to office each day. He never

gin
goes 2 days in a row by train but if he drives one day, then the next day he is just
as likely to drive again as he is to travel by train. Now suppose that on the first

ee
day of the week, the man tossed a fair die and drove to work iff a 6 appeared.

rin
Find (1) the probability that he takes a train on the 3 rd day. (2) the probability that
he drives to work in the long run.

g.n
11. Show that the process X (t) = A cost + B sin t (where A and B are random

et
variables) is wide sense stationary, if (1) E (A ) = E(B) = 0 (2) E(A2) = E (B2 ) and
E(AB) = 0
12. Find probability distribution of Poisson process.
13. Prove that sum of two Poisson processes is again a Poisson process.
14. Write classifications of random processes with example

Unit 4
Correlation and Spectrum Densities

Downloaded From : www.EasyEngineering.net


Downloaded From : www.EasyEngineering.net

WWW.VIDYARTHIPLUS.COM

ww
w.E
asy
En
gin
ee rin
g.n
et

V+ TEAM

Downloaded From : www.EasyEngineering.net


Downloaded From : www.EasyEngineering.net

WWW.VIDYARTHIPLUS.COM

ww
w.E
asy
En
gin
ee rin
g.n
et

V+ TEAM

Downloaded From : www.EasyEngineering.net


Downloaded From : www.EasyEngineering.net

WWW.VIDYARTHIPLUS.COM

ww
w.E
asy
En
gin
ee rin
g.n
et

Downloaded From : www.EasyEngineering.net


Downloaded From : www.EasyEngineering.net

WWW.VIDYARTHIPLUS.COM

ww
w.E
asy
En
gin
ee rin
g.n
et

Downloaded From : www.EasyEngineering.net


Downloaded From : www.EasyEngineering.net

ww
w.E
asy
En
gi nee
rin
g.n
et

Downloaded From : www.EasyEngineering.net


Downloaded From : www.EasyEngineering.net

ww
w.E
asy
En
gi nee
rin
g.n
et

Downloaded From : www.EasyEngineering.net


Downloaded From : www.EasyEngineering.net

ww
w.E
asy
En
gi nee
rin
g.n
et

Downloaded From : www.EasyEngineering.net


Downloaded From : www.EasyEngineering.net

ww
w.E
asy
En
gi nee
rin
g.n
et

Downloaded From : www.EasyEngineering.net


Downloaded From : www.EasyEngineering.net

B.E./B.Tech. DEGREE EXAMINATION, APRIL/MAY 2010


Fourth Semester

Electronics and Communication Engineering

MA2261 – PROBABILITY AND RANDOM PROCESS

(Common to Bio – Medical Engineering)

(Regulation 2008)

Time : Three hours Maximum : 100 marks

ww Use of Statistical Tables is permitted

w.E Answer ALL Questions

PART A – (10 x 2 = 20 Marks)

asy
1. If the p.d.f of a random variable X is f ( x ) =
x
2
in 0 ≤ x ≤ 2 , find P ( X > 1.5 / X > 1) .

En (
1 5t
e − e 4 t ) , find E ( X ) .
gin
2. If the MGF of a uniform distribution for a random variable X is
t

to be the joint density function. ee


3. Find the value of k , if f ( x , y ) = k (1 − x )(1 − y ) in 0 < x , y < 1 and f ( x , y ) = 0 , otherwise, is

rin
4. A random variable X has mean 10 and variance 16. Find the lower bound for P (5 < X < 15) .

5. Define a wide sense stationary process. g.n


6. Define a Markov chain and give an example. et
7. Find the mean of the stationary process { x ( t )} , whose autocorrelation function is given by
9
R(τ ) = 16 + .
1 + 16τ 2

8. Find the power spectral density function of the stationary process whose autocorrelation
−τ
function is given by e .

9. Define time – invariant system.

10. State autocorrelation function of the white noise.

Downloaded From : www.EasyEngineering.net


Downloaded From : www.EasyEngineering.net

PART B – (5 x 16 = 80 marks)

11. (a) (i) The probability mass function of random variable X is defined as P ( X = 0) = 3C 2 ,

P ( X = 1) = 4C − 10C 2 , P ( X = 2) = 5C − 1 , where C > 0 and P ( X = r ) = 0 if

r ≠ 0,1, 2 . Find

(1) The value of C

(2) P (0 < X < 2 / x > 0)

ww (3) The distribution function of X

w.E (4) The largest value of X for which F ( x ) <


1
2
.

asy
(ii) If the probability that an applicant for a driver’s license will pass the road test on any

En
given trial is 0.8. What is the probability that he will finally pass the test

gin
(1) On the fourth trial and

(2) In less than 4 trials?

ee
Or

rin
(b) (i) Find the MGF of the two parameter exponential distribution whose density function is

g.n
given by f ( x ) = λ e − λ ( x − a ) , x ≥ a and hence find the mean and variance.

et
(ii) The marks obtained by a number of students in a certain subject are assumed to be

normally distributed with mean 65 and standard deviation 5. If 3 students are selected at

random from this group, what is the probability that two of them will have marks over 70?

12. (a) (i) Find the bivariate probability distribution of (X,Y) given below:

Downloaded From : www.EasyEngineering.net


Downloaded From : www.EasyEngineering.net

Y 1 2 3 4 5 6

0 0 0 1/32 2/32 2/32 3/32

1 1/16 1/16 1/8 1/8 1/8 1/8

2 1/32 1/32 1/64 1/64 0 2/64

Find the marginal distributions, conditional distribution of X given Y = 1 and conditional


distribution of Y given X = 0.

ww (ii) Find the covariance of X and Y, if the random variable (X,Y) has the joint p.d.f

w.E f ( x , y ) = x + y , 0 ≤ x ≤ 1, 0 ≤ y ≤ 1 and f ( x , y ) = 0 , otherwise.

asy
Or

8
(b) (i) The joint p.d.f of two dimensional random variable (X,Y) is given by f ( x , y ) =

En 9
xy ,

gin
0 ≤ x ≤ y ≤ 2 and f ( x , y ) = 0 , otherwise. Find the densities of X and Y, and the

ee
conditional densities f ( x / y ) and f ( y / x ) .

rin
(ii) A sample of size 100 is taken from a population whose mean is 60 and variance is 400.

g.n
Using Central Limit Theorem, find the probability with which the mean of the sample will

not differ from 60 by more than 4.

et
13. (a) (i) Examine whether the random process { X ( t )} = A cos(ω t + θ ) is a wide sense
stationary if A and ω are constants and θ is uniformly distributed random variable in (0,2π).

(ii) Assume that the number of messages input to a communication channel in an interval
of duration t seconds, is a Poisson process with mean λ = 0.3 . Compute

(1) The probability that exactly 3 messages will arrive during 10 second interval

(2) The probability that the number of message arrivals in an interval of duration 5
seconds is between 3 and 7.

Or

Downloaded From : www.EasyEngineering.net


Downloaded From : www.EasyEngineering.net

(b) (i) The random binary transmission process { X ( t )} is a wide sense process with zero mean
τ
and autocorrelation function R(τ ) = 1 − , where T is a constant. Find the mean and variance
T
of the time average of { X ( t )} over (0, T). Is { X ( t )} mean – ergodic?

(ii) The transition probability matrix of a Markov chain { X ( t )} , n = 1, 2, 3, ... having three

 0.1 0.5 0.4 


 
states 1, 2, 3 is P =  0.6 0.2 0.2  , and the initial distribution is P (0) = [ 0.7 0.2 0.1] , Find
 0.3 0.4 0.3 

P ( X 2 = 3 ) and P ( X 3 = 2, X 2 = 3, X 1 = 3, X 0 = 2 ) .

ww
14. (a) (i) Find the autocorrelation function of the periodic time function of the period time

w.E function { X ( t )} = A sin ω t .

asy
(ii) The autocorrelation function of the random binary transmission { X ( t )} is given by

R(τ ) = 1 −
τ
En
for τ < T and R(τ ) = 0 for τ < T . Find the power spectrum of the

gin
T

process { X ( t )} .

Oree rin
{ X ( t )} and {Y ( t )} are zero mean and stochastically independent random processes
(b) (i)

having autocorrelation functions RXX (τ ) = e


−τ
g.n
and RYY (τ ) = cos 2πτ respectively. Find

et
(1) The autocorrelation function of W ( t ) = X ( t ) + Y ( t ) and Z ( t ) = X ( t ) − Y ( t )

(2) The cross correlation function of W ( t ) and Z ( t ) .

(ii) Find the autocorrelation function of the process { X ( t )} for which the power spectral

density is given by S XX (ω ) = 1 + ω 2 for ω < 1 and S XX (ω ) = 0 for ω > 1 .

15. (a) (i) A wide sense stationary random process { X ( t )} with autocorrelation RXX (τ ) = e
−aτ

where A and a are real positive constants, is applied to the input of an Linear transmission input
system with impulse response h( t ) = e − bt u( t ) where b is a real positive constant. Find the
autocorrelation of the output Y ( t ) of the system.

Downloaded From : www.EasyEngineering.net


Downloaded From : www.EasyEngineering.net

(ii) If { X ( t )} is a band limited process such that S XX (ω ) = 0 when ω > σ , prove that

2 RXX (0) − RXX (τ ) ≤ σ 2τ 2 RXX (0) .

Or

(b) (i) Assume a random process X ( t ) is given as input to a system with transfer function
N0
H (ω ) = 1 for −ω 0 < ω < ω 0 . If the autocorrelation function of the input process is δ (t ) ,
2
find the autocorrelation function of the output process.

(ii) If Y ( t ) = A cos(ω t + θ ) + N ( t ) , where A is a constant, θ is a random variable with a

ww
uniform distribution in ( −π , π ) and { N ( t )} is a band limited Gaussian white noise with a

w.E
N0
power spectral density S NN (ω ) = for ω − ω 0 < ω B and S NN (ω ) = 0 , elsewhere. Find the
2
power spectral density of Y ( t ) , assuming that N ( t ) and θ are independent.

asy
En
gin
ee rin
g.n
et

Downloaded From : www.EasyEngineering.net


Downloaded From : www.EasyEngineering.net

B.E./B.Tech. DEGREE EXAMINATIONS, APRIL/MAY 2011


Fourth Semester 

Electronics and Communication Engineering  

MA2261 – PROBABILITY AND RANDOM PROCESS

(Common to Bio – Medical Engineering) 

(Regulation 2008) 

Time : Three hours                Maximum : 100 marks 

ww Answer ALL Questions 

w.E PART A – (10 x 2 = 20 Marks) 

1. The CDF of a continuous random variable is given by  F ( x ) = 
 0, x<0

asy − x/5
.  Find the 
1 − e , 0≤ x<∞
PDF and mean of X . 

En
2. Establish the memoryless property of the exponential distribution. 

gin
3. Let  X and  Y be continuous random variables with joint probability density function 

f XY ( x , y ) =

fY / X ( y / x ) . 
x( x − y )
8 ee
, 0 < x < 2, − x < y < x  and  f XY ( x , y ) = 0  elsewhere. Find 

rin
g.n
4. Find the acute angle between the two lines of regression, assuming the two lines of regression. 

5. Prove that a first order stationary process has a constant mean. 

6. State the postulates of a Poisson process. 
et
9
7. The autocorrelation function of a stationary random process is R(τ ) = 16 + . Find the 
1 + 16τ 2
mean and variance of the process. 

8. Prove that for a WSS process  { X ( t )} , RXX ( t , t + τ )  is an even function of  τ . 

9. Find the system Transfer function, if a Linear Time Invariant system has an impulse function 
1
 ;t ≤c
H ( t ) =  2c . 
0 ; t ≥ c

Downloaded From : www.EasyEngineering.net


Downloaded From : www.EasyEngineering.net

10. Define white noise. 

PART B – (5 x 16 = 80 marks) 

11.   (a)  The probability density function of a random variable  X  is given by     

 x, 0< x<1

                 f X ( x ) =  k (2 − x ), 1 ≤ x ≤ 2 . 
 0, otherwise

(1) Find the value of ‘ k ’.              (4) 

ww (2) Find  P (0.2 < x < 1.2)               (3) 

w.E (3) What is  P [ 0.5 < x < 1.5 / x ≥ 1]           (4) 

asy
(4) Find the distribution function of f ( x ) .          (5) 

     

En Or 

gin
  (b)    (i)  Derive the m.g.f of Poisson distribution and hence or otherwise deduce its mean and  

variance. 

 
ee
       (ii)  The marks obtained by a number of students in a certain subject are assumed to be   

rin
        normally distributed with mean 65 and standard deviation 5.  If 3 students are selected at   

g.n
        random from this set, what is the probability that exactly 2of them will have marks over 70? 

et
12.   (a) (i)  If  X and  Y are independent Poisson random variables with respective parameters  λ1   

and  λ2 . Calculate the conditional distribution of  X , given that  X + Y = n  . 

     (ii) The regression equation of  X on  Y is  3Y − 5 X + 108 = 0 . If the mean value of  Y  is 44  

and the variance of  X is 9/16th of the variance of  Y . Find the mean value of  X and the  

correlation coefficient. 

        Or 

Downloaded From : www.EasyEngineering.net


Downloaded From : www.EasyEngineering.net

(b)   (i) If  X and  Y are independent random variables with density function 


y
1, 1 ≤ x ≤ 2  , 2≤ y≤4
f X ( x) =   and  fY ( y ) =  6 , find the density function of 
 0, otherwise  0, otherwise
Z = XY . 

        (ii)  The life time of a particular variety of electric bulb may be considered as a random  

variable with mean 1200 hours and standard deviation 250 hours. Using central limit  

theorem, find the probability that the average life time of 60 bulbs exceeds 1250 hours. 

13.   (a)   (i)  A random process X ( t ) defined by  X ( t ) = A cos t + B sin t , − ∞ < t < ∞ , where   

ww A and B are independent random variables each of which takes a value  −2  with  

w.E probability  1 / 3 and a value  1 with probability  2 / 3 . Show that  X ( t ) is wide – sense  

stationary. 
asy
        (ii)  A random process has sample functions of the form X ( t ) = A cos ( ω t + θ ) , where   

En
ω is constant,  A is a random variable with mean zero and variance one and  θ is a  
gin
random variable that is uniformly distributed between  0 and  2π . Assume that the  

       
ee
random variables  A and  θ are independent. Is  X ( t ) is a mean – ergodic process? 

Or  rin
g.n
(b)  (i)  If  { X ( t )} is a Gaussian process with  µ ( t ) = 10  and  C ( t1 , t 2 ) = 16e
− t1 − t 2
, find the  

probability that   et
(1) X (10) ≤ 8  

(2) X (10) − X (6) ≤ 4  

(ii)  Prove that the interval between two successive occurrences of a Poisson process with  

1
       parameter λ has an exponential distribution with mean  . 
λ

Downloaded From : www.EasyEngineering.net


Downloaded From : www.EasyEngineering.net

14. (a)   (i)  The power spectral density function of a zero mean WSS process  X ( t ) is given by 


1, ω < ω 0  π 
S (ω ) =  . Find R(τ ) and show that  X ( t ) and X  t +  are uncorrelated. 
 0, otherwise  ω0 
−2 λ τ
        (ii)  The Auto correlation function of a WSS process is given by  R(τ ) = α e
2
determine   

the power spectral density of the process. 

Or 

(b)   (i)   State and prove Weiner – Khintchine Theorem. 

ww         (ii)  The cross – power spectrum of real random processes  { X ( t )} and  {Y ( t )} is given by  

 a + bjω , for ω < 1


w.E S xy (ω ) = 
 0, elsewhere
. Find the cross correlation function. 

asy
15. (a)  (i)  Consider a system with transfer function 
1
1 + jω
. An input signal with autocorrelation   

En
function  mδ (τ ) + m 2 is fed as input to the system. Find the mean and mean – square  

value of the output.  gin
ee
        (ii)  A stationary random process  X ( t ) having the autocorrelation function  

rin
RXX (τ ) = Aδ (τ ) is applied to a linear system at time  t = 0 where  f (τ ) represent the  

g.n
impulse function. The linear system has the impulse response of  h( t ) = e − bt u( t ) where  

et
u( t ) represents the unit step function. Find  RYY (τ ) . Also find the mean and variance of  

Y ( t ) . 

                Or 

−t
1 RC
(b)    (i)  A linear system is described by the impulse response  h( t ) = e u( t ) . Assume an  
RC

input process whose Auto correlation function is  Bδ (τ ) . Find the mean and Auto  

correlation function of the output process. 

Downloaded From : www.EasyEngineering.net


Downloaded From : www.EasyEngineering.net

         (ii) If  { N ( t )} is a band limited white noise centered at a carrier frequency  ω 0  such that  

 N0
 , for ω − ω0 < ω B
S NN (ω ) =  2 . Find the autocorrelation of  { N ( t )} . 
 0, elsewhere

ww
w.E
asy
En
gin
ee rin
g.n
et

Downloaded From : www.EasyEngineering.net


Downloaded From : www.EasyEngineering.net

ww
w.E
asy
En
gin
ee rin
g.n
et

Downloaded From : www.EasyEngineering.net


Downloaded From : www.EasyEngineering.net

ww
w.E
asy
En
gin
ee rin
g.n
et

Downloaded From : www.EasyEngineering.net


Downloaded From : www.EasyEngineering.net

ww
w.E
asy
En
gin
ee rin
g.n
et

Downloaded From : www.EasyEngineering.net


Downloaded From : www.EasyEngineering.net

ww
w.E
asy
En
gin
ee rin
g.n
et

Downloaded From : www.EasyEngineering.net


Downloaded From : www.EasyEngineering.net

ww
w.E
asy
En
gin
ee rin
g.n
et

Downloaded From : www.EasyEngineering.net


Downloaded From : www.EasyEngineering.net

ww
w.E
asy
En
gin
ee rin
g.n
et

Downloaded From : www.EasyEngineering.net


Downloaded From : www.EasyEngineering.net

ww
w.E
asy
En
gin
ee rin
g.n
et

Downloaded From : www.EasyEngineering.net


Downloaded From : www.EasyEngineering.net

ww
w.E
asy
En
gin
ee rin
g.n
et

Downloaded From : www.EasyEngineering.net


Downloaded From : www.EasyEngineering.net

ww
w.E
asy
En
gin
ee rin
g.n
et

Downloaded From : www.EasyEngineering.net


Downloaded From : www.EasyEngineering.net

ww
w.E
asy
En
gin
ee rin
g.n
et

Downloaded From : www.EasyEngineering.net


Downloaded From : www.EasyEngineering.net

ww
w.E
asy
En
gin
ee rin
g.n
et

Downloaded From : www.EasyEngineering.net


Downloaded From : www.EasyEngineering.net

ww
w.E
asy
En
gin
ee rin
g.n
et

Downloaded From : www.EasyEngineering.net

Potrebbero piacerti anche